Sei sulla pagina 1di 70

Ecuaciones Diferenciales Ordinarias

Problemas resueltos y propuestos

Joe Garca
Copyright c 2017 Joe Garca

P UBLISHED BY P UBLISHER

BOOK - WEBSITE . COM

Licensed under the Creative Commons Attribution-NonCommercial 3.0 Unported License (the
License). You may not use this file except in compliance with the License. You may obtain a
copy of the License at http://creativecommons.org/licenses/by-nc/3.0. Unless required
by applicable law or agreed to in writing, software distributed under the License is distributed on an
AS IS BASIS , WITHOUT WARRANTIES OR CONDITIONS OF ANY KIND, either express or implied.
See the License for the specific language governing permissions and limitations under the License.

First printing, March 2017


1. Aplicaciones

Consideramos los problemas que dan lugar a algunos de los tipos de ecuaciones diferenciales
ordinarias de primer orden estudiadas en el los captulos anteriores. En primer lugar, formulamos
el problema matemticamente, obteniendo as una ecuacin diferencial. Entonces resolvemos la
ecuacin e intentamos interpretar la solucin en trminos de las cantidades involucradas en el
problema original.

1.1 Soluciones singulares


Suponga que la ecuacin diferencial

F x, y, y 0 = 0 (1.1)

tenga la integral general

F(x, y, C) = 0 (1.2)

Suponga tambin que la familia de las curvas integrales, correspondiente a la ecuacin (1.2), tiene
una envolvente, sta es tambin una curva integral de la ecuacin diferencial (1.1).
La envolvente toca en cada uno de sus puntos a cierta curva de la familia, es decir, la envolvente
tiene en este punto una tangente comn con la curva. Por consiguiente, en cada punto comn la
envolvente y la curva de la familia tienen valores iguales de las magnitudes x, y, y 0 . Pero, para la
curva de la familia las magnitudes x, y, y 0 satisfacen la ecuacin (1.1).
Por tanto, la abscisa, la ordenada y el coeficiente angular de cada punto de la envolvente
satisfacen tambin la misma ecuacin lo que significa que la envolvente es una curva integral y
que su ecuacin es una solucin de la ecuacin diferencial dada. Pero, como la envolvente no es
en general una curva de la familia, su ecuacin no se puede deducir de la integral general (1.2),
cualquiera que sea el valor particular de C. La solucin de la ecuacin diferencial que no se obtiene
de la integral general cualquiera que sea el valor de C, y que tiene como grfica la envolvente de
la familia de curvas integrales que entran en la solucin general, se llama solucin singular de la
ecuacin diferencial.
4 Captulo 1. Aplicaciones

Suponemos conocida la integral general

Y(x, y, C) = 0

eliminando C de esta ecuacin y de la ecuacin Y C0 (x, y, C) = 0 obtenemos la ecuacin y(x, y) = 0.


Si esta funcin satisface la ecuacin diferencial (y no pertenece a la familia (1.2)), entonces es la
integral singular.

R Ntese que por todo punto de la curva que represente la solucin singular pasan por lo menos
dos curvas integrales, es decir, la unicidad de la solucin se perturba en cada punto de una
solucin singular.

Ejemplo 1.1 Hallar las soluciones singulares, si stas existen:

(xy 0 + y)2 + 3x5 (xy 0 2y) = 0.

Solucin Establecemos un sistema de ecuaciones, entre la ecuacin original y su derivada


( (
(xy 0 + y)2 + 3x5 (xy 0 2y) = 0 (xy 0 + y)2 + 3x5 (xy 0 2y) = 0
)
2(xy 0 + y)x + 3x5 x = 0 xy 0 + y = 32 x5

eliminamos la variable y 0
2
3 5 5 3 5 9 10
x + 3x x 3y = 0 ) x + 9x5 y = 0
2 2 4

(
9 5 5 x=0 1 5
x (x + 4y) = 0 ) ) y= x
4 y = 14 x5 4

Reemplazando en la ecuacin original, comprobamos que es solucin de la ecuacin. Por tanto, la


ecuacin encontrada es solucin singular.

Ejemplo 1.2 Hallar las soluciones singulares, si stas existen:

(1 + y 0 2 )y2 4yy 0 4x = 0.

Solucin Establecemos un sistema de ecuaciones, entre la ecuacin original y su derivada


( (
(1 + y 0 2 )y2 4yy 0 4x = 0 (xy 0 + y)2 + 3x5 (xy 0 2y) = 0
)
2y2 y 0 4y = 0 xy 0 + y = 32 x5

eliminamos la variable y 0
( p
4 2 y1 = 2 x + 1
1 + 2 y2 4x 4x = 0 ) 2
y = 4x + 4 ) p
y y y2 = 2 x + 1

Para determinar si estos ramales son o no soluciones singulares, hallamos la envolvente de la


familia, es decir la curva integral
( p
y1 = 12 4cx c2 4(x2 4x 4)
1
p
y2 = 2 4cx c2 4(x2 4x 4)
1.2 Ejercicios 5

derivamos con respecto de c


4x 2c = 0 ) c = 2x,
reemplazando, tenemos
4y2 = 4( 2x)x (2x)2 4(x2 4x 4) ) y2 = 4x + 4,
esta ecuacin coincide con la encontrada, por lo tanto los ramales son soluciones singulares de la
ecuacin propuesta.

1.2 Ejercicios
1. Encuentra las soluciones singulares de las siguientes ecuaciones diferenciales dadas:
a) y 0 2 + y2 1 = 0; Resp: .

b) x2 y 0 2 + (2x y)yy 0 + y2 = 0; Resp: .

c) y0 3 4xyy 0 + 8y2 = 0; Resp: .

1 2
d) y0 2 xy 0 y+ x = 0; Resp: .
2

1.3 Trayectorias ortogonales y oblicuas


1.3.1 Trayectorias ortogonales
Definicin 1.3.1 Sea

F(x, y, c) = 0 (1.3)

una familia dada de curvas paramtricas en el plano xy. Una curva que interseca las curvas de la
familia (1.3) en ngulos rectos se denomina trayectoria ortogonal de la familia dada.

El problema de encontrar las trayectorias ortogonales de una familia dada de curvas surge en
muchas situaciones fsicas. Por ejemplo, en un campo elctrico bidimensional las lneas de fuerza
(lneas de flujo) y las curvas equipotenciales son trayectorias ortogonales entre s.
Ahora procedemos a encontrar las trayectorias ortogonales de una familia de curvas (1.3). Se
obtiene la ecuacin diferencial de la familia (1.3), diferenciando la ecuacin (1.3) implcitamente
con respecto a x y luego eliminando el parmetro c entre la ecuacin derivada obtenida y la ecuacin
dada (1.3). Suponemos que la ecuacin diferencial resultante de la familia (1.3) puede expresarse
en la forma
y 0 = f (x, y). (1.4)
As, la curva de C de la familia dada (1.3) que pasa a travs del punto (x, y) tiene la pendiente
f (x, y) all. Dado que una trayectoria ortogonal de la familia dada intersecta cada curva de la
familia en ngulos rectos, la pendiente de la trayectoria ortogonal a C a (x, y) es
1
.
f (x, y)
As, la ecuacin diferencial de la familia de trayectorias ortogonales es
1
y0 = . (1.5)
f (x, y)
6 Captulo 1. Aplicaciones

Una familia de un solo parmetro

G(x, y, c) = 0

y = F(x, c)

de soluciones de las ecuaciones diferenciales (1.5) representa la familia de trayectorias ortogonales


de la familia original (1.3), excepto posiblemente para ciertas trayectorias que son lneas verticales.

Resumimos este procedimiento de la siguiente manera:


Paso 1. De la ecuacin

F(x, y, c) = 0 (1.6)

de la familia dada de curvas, encuentre la ecuacin diferencial

y 0 = f (x, y) (1.7)

de esta familia.
Paso 2. En la ecuacin diferencial y 0 = f (x, y) que se encuentra en el Paso 1, reemplace f (x, y)
1
por su recproco negativo . Esto da la ecuacin diferencial
f (x, y

1
y0 = (1.8)
f (x, y)

de las trayectorias ortogonales.


Paso 3. Obtener una familia de un parmetro

G(x, y, c) = 0 o y = F(x, c)

de soluciones de la ecuacin diferencial (1.11), obteniendo as la familia deseada de trayectorias


ortogonales (excepto posiblemente para ciertas trayectorias que son lneas verticales y deben ser
determinadas por separado).

En el Paso 1, al encontrar la ecuacin diferencial (1.10) de la familia dada, asegrese de eliminar


el parmetro c durante el proceso.

Ejemplo 1.3 Hallar la familia de trayectorias ortogonales a la familia de curvas dada:

4y + x2 + 1 Ce2y = 0.

Solucin

C = (4y + x2 + 1)e 2y

x dx + (1 4y x2 ) dy = 0
1
Como y 0 = y0

( 1 + 4y + x2 ) dx + x dy = 0
1.3 Trayectorias ortogonales y oblicuas 7

La ecuacin diferencial no es exacta, por tanto el factor integrante es = x3 y

x3 ( 1 + 4y + x2 ) dx + x4 dy = 0

se convierte en una ecuacin exacta. Buscamos la solucin


Z Z Z
3 2
F(x, y) = x ( 1 + 4y + x ) x + x4 x3 ( 1 + 4y + x2 ) x dy
y
Z
1 4 2 1 4 2
= x (2x + 12y 3) + ln(x2 + 1) + x4 x (2x + 12y 3) dy
12 y 12
Z
1 4 2
= x (2x + 12y 3) + x4 x4 dy
12
1 4 2
= x (2x + 12y 3).
12
Como F(x, y) = c, entonces la solucin de la ecuacin diferencial es
1 4 2
x (2x + 12y 3) = c,
12
y la familia de trayectorias ortogonales es

(2x2 + 12y 3)x4 = c.

Ejemplo 1.4 Hallar el miembro de la familia de trayectorias ortogonales a la familia de curvas


x + y = Cey , que pasa por el punto (0, 5).
Solucin
y
C = (x + y)e
1
(x + y 1)y 0 = 1 ) y0 =
x+y 1
1
Como y0 = y0 , entonces

1 1
= ) y0 +y = 1 x
y 0 x+y 1

R
dx x
v(x) = e =e

Z
1 x
u(x) = dx + c = (2 x)ex + c
e x

y = [c + (2 x)ex ]e x
) y = ce x
x+2

reemplazando el punto (0, 5) en la ecuacin, obtenemos c = 3. Por lo tanto el miembro de la


familia de trayectorias ortogonales es
x
y = 3e x + 2.


8 Captulo 1. Aplicaciones

Ejemplo 1.5 Hallar la familia de trayectorias ortogonales a la familia de curvas dada:

y2 x2 + 4xy 2Cx = 0.

Solucin
y2 x2 + 4xy
C=
2x
x2 + y2
y0 =
2xy + 4x2
1
Como y 0 = y0

2xy + 4x2
y0 =
x 2 + y2

(2xy + 4x2 ) dx + (x2 + y2 ) dy = 0

Realizamos la prueba de exactitud


M
M = 2xy + 4x2 ) = 2x
y
N
N = x 2 + y2 ) = 2x
x
M N
puesto que y = x , la ecuacin diferencial es exacta. Utilizando la frmula (??), encontramos
F(x, y):
Z Z Z

F(x, y) = 2xy + 4x2 x + x2 + y2 2xy + 4x2 x dy
y
Z
2 4 3 2 2 2 4 3
= x y+ x + x +y x y+ x dy
3 y 3
Z
4
= x2 y + x3 + x2 + y2 x2 dy
3
Z
4
= x2 y + x3 + y2 dy
3
4 1
= x2 y + x3 + y3 .
3 3
Como F(x, y) = c, entonces la solucin general de la ecuacin diferencial es

3x2 y + 4x3 + y3 = c.

1.3.2 Trayectorias oblicuas


Definicin 1.3.2 Sea

F(x, y, c) = 0 (1.9)

una familia de curvas de un solo parmetro. Una curva que interseca las curvas de la familia
(1.9) en un ngulo constante a 6= 90 se denomina trayectoria oblicua de la familia dada.
1.4 Envolvente de una familia de curvas 9

Supongamos que la ecuacin diferencial de una familia es

y 0 = f (x, y). (1.10)

Entonces la curva de la familia (1.10) a travs del punto (x, y) tiene una pendiente f (x, y) a (x, y) y,
su lnea tangente tiene un ngulo de inclinacin arctan[ f (x, y)]. La lnea tangente de una trayectoria
oblicua que interseca esta curva en el ngulo a tendr as de inclinacin

arctan[ f (x, y)] + a

en el punto (x, y). Por lo tanto, la pendiente de esta trayectoria oblicua es dada por

f (x, y) + tan a
tan {arctan[ f (x, y)] + a} = .
1 f (x, y) tan a

As, la ecuacin diferencial de tal familia de trayectorias oblicuas viene dada por

f (x, y) + tan a
y0 = .
1 f (x, y) tan a

As, para obtener una familia de trayectorias oblicuas que intersecan una familia dada de curvas
en el ngulo constante a 6= 90 , podemos seguir los tres pasos del procedimiento anterior para
encontrar las trayectorias ortogonales, excepto que sustituimos el Paso 2 por el siguiente paso:
Paso 2. En la ecuacin diferencial y 0 = f (x, y) de la familia dada, reemplace f (x, y) por la expresin

f (x, y) + tan a
. (1.11)
1 f (x, y) tan a

1.4 Envolvente de una familia de curvas


Sea dada una ecuacin de la forma

F(x, y, C) = 0, (1.12)

donde x e y son coordenadas variables y C, un parmetro que pueda tomar diferentes valores fijados.
Para cada valor dado del parmetro C la ecuacin (1.12) determina cierta curva en el plano XY .
Dando a C todos los valores posibles, obtenemos una familia de curvas que dependen de un slo
parmetro, o, una familia de curvas monoparamtricas (puesto que contiene una sola constante
arbitraria).
Definicin 1.4.1 La lnea L se llama envolvente de una familia de curvas monoparamtricas, si
en cada uno de sus puntos toca una u otra curva de la familia, y tambin, diferentes curvas de la
familia dada tocan la lnea L en distintos puntos.

Sea la familia de curvas

F(x, y, C) = 0, (1.13)

que dependen de un parmetro C. Supongamos que esta familia tenga una envolvente cuya ecuacin
se puede escribir en la forma y = j(x), donde j(x) es una funcin continua y derivable de x.
Examinemos un punto M(x, y) que se halla en la envolvente. Este punto pertenece, tambin, a
cierta curva de la familia (1.13). A esta curva corresponde un valor determinado del parmetro C,
10 Captulo 1. Aplicaciones

este valor para dadas (x, y) se define de la ecuacin (1.13), C = C(x, y). Por tanto, para todos los
puntos de la envolvente se verifica la igualdad

F(x, y, C(x, y)) = 0. (1.14)

Supongamos que C(x, y) sea funcin derivable, no constante en ningn intervalo de los valores
estudiados de x, y.

Partiendo de la ecuacin (1.14) de la envolvente, encontremos el coeficiente angular de la tan-


gente a la envolvente en el punto M(x, y). Derivamos la ecuacin (1.14) respecto a x, considerando
y como funcin de x:

F F C F F C 0
+ + + y =0
x C x y C y


C C 0
F 0x + F 0y y 0
+ FC0 + y = 0. (1.15)
x y

Luego, el coeficiente angular de la tangente a la curva de la familia (1.13) en el punto M(x, y) se


deduce de la ecuacin

F 0x + F 0y y 0 = 0 (1.16)

en la curva dada C es constante. Supngase que F 0y 6= 0; en caso contrario consideremos x como la


funcin e y, como el argumento. Puesto que el coeficiente angular k de la envolvente es igual al de
la curva de la familia, de las ecuaciones (1.15) y (1.16) se deduce

0 C C 0
FC + y = 0.
x y

Pero como C(x, y) 6= cte en la envolvente, entonces

C C 0
+ y 6= 0,
x y

por lo que para sus puntos es vlida la igualdad

F C0 (x, y,C) = 0 (1.17)

As, para determinar la envolvente sirven las dos ecuaciones siguientes


(
F(x, y, C) = 0
(1.18)
F C0 (x, y, C) = 0

eliminando C de estas ecuaciones, obtenemos y = j(x), donde j(x) es una funcin derivable,
siendo C 6= cte en esta curva, entonces y = j(x) es la ecuacin de la envolvente.

R Si una funcin y = j(x) es la ecuacin del lugar geomtrico de puntos singulares de la familia
(1.13), es decir, de los puntos donde Y 0x = 0 y Y 0y = 0, entonces las coordenadas de estos
puntos tambin satisfacen las ecuaciones (1.18).
1.5 Ejercicios 11

1.5 Ejercicios
1. En los siguientes ejercicios encuentre las trayectorias ortogonales de cada familia dada de
curvas. En cada caso bosqueje varios miembros de la familia y varias de las trayectorias
ortogonales en el mismo conjunto de ejes:
a) y = cx3 ; Resp: .

b) y2 = cx; Resp: .

c) cx2 + y2 = 1; Resp: .

d) y = ecx ; Resp: .

e) y=x 1 + ce x ; Resp: .

f) (x + 1)y = cx2 ; Resp: .

g) x2 + y2 = cx3 ; Resp: .

h) x2 = 2y 1 + ce 2y
; Resp: .

i) 4xy2 = y4 + 4c; Resp: .

j) x2 y2 = cx3 ; Resp: .

2. Encuentre las trayectorias ortogonales de la familia de elipses que tienen centro en el origen,
un foco en el punto (c, 0), y el eje semimayor de longitud 2c.
Resp: .

3. Encuentre las trayectorias ortogonales de la familia de crculos que son tangentes al eje y en
el origen.
Resp: .

4. Encuentre el valor de k tal que las parbolas y = c1 x2 + k sean las trayectorias ortogonales de
la familia de elipses x2 + 2y2 y = c2 .
Resp: .

5. Encuentre el valor de n tal que las curvas xn + yn = c1 sean las trayectorias ortogonales de la
familia (1 c2 x)y = x.
Resp: .

6. Se dice que una familia dada de curvas es auto-ortogonal si su familia de trayectorias ortogo-
nales es la misma que la familia dada. Demuestre que la familia de parbolas y2 = 2cx + c2
es auto-ortogonal.
Resp: .

7. Encuentre una familia de trayectorias oblicuas que intersecan la familia de crculos x2 + y2 =


c2 en el ngulo 45 .
Resp: .
12 Captulo 1. Aplicaciones

8. Encuentre una familia de trayectorias oblicuas que intersecan la familia de parbolas y2 = cx


en el ngulo 60 .
Resp: .

9. Encuentre una familia de trayectorias oblicuas que intersecan la familia de curvas x + y = cx2
en el ngulo a tal que tan a = 2.
Resp: .

10. Encuentre las trayectorias ortogonales de la familia de curvas y = c x + 1x .


Resp: .

11. Encuentre las trayectorias ortogonales de la familia de curvas x3 + 3xy2 = c.


Resp: .

1.6 Problemas en Mecnica


Antes de aplicar nuestro conocimiento de las ecuaciones diferenciales a ciertos problemas de
la mecnica, recordemos brevemente ciertos principios de ese sujeto. El impulso de un cuerpo se
define como el producto mv de su masa m y su velocidad v. La velocidad v y, por tanto, el momento
son cantidades vectoriales. Ahora declaramos la siguiente ley bsica de la mecnica:
Segunda ley de Newton. La velocidad de cambio de momentum de un cuerpo es proporcional
a la fuerza resultante que acta sobre el cuerpo y est en la direccin de esta fuerza resultante.
En el lenguaje matemtico, esta ley establece que
d
(mv) = KF,
dt
donde m es la masa del cuerpo, v es su velocidad, F es la fuerza resultante que acta sobre l, y K
es una constante de proporcionalidad. Si la masa m se considera constante, esto se reduce a
dv
m = KF,
dt
o
K
a=K , (1.19)
m
o

F = kma, (1.20)

donde k = K1 y a = dv
dt es la aceleracin del cuerpo. La forma (1.19) es una declaracin matemtica
directa de la manera en que la segunda ley de Newton se expresa generalmente en palabras, siendo
la masa considerada constante. Sin embargo, utilizaremos la forma equivalente (1.20). La magnitud
de la constante de proporcionalidad k depende de las unidades empleadas para la fuerza, la masa
y la aceleracin. Obviamente, los sistemas ms simples de unidades son aquellos para los cuales
k = 1. Cuando se utiliza tal sistema (1.20) se reduce a

F = ma, (1.21)

Es en esta forma que usaremos la segunda ley de Newton. Observe que la ecuacin (1.21) es una
ecuacin vectorial.
1.6 Problemas en Mecnica 13

Sistema britnico [cgs] Sistema [mks] Sistema


fuerza libra dina newton
masa slug gramo kilogramo
distancia pie centmetro metro
tiempo segundo segundo segundo
aceleracin [pie/sec2 ] [cm/seg2 ] [m/seg2 ]

Cuadro 1.1: Unidades de los tres sistemas

Varios sistemas de unidades para las cuales k = 1 estn en uso. En este texto usaremos slo
tres: el sistema gravitatorio britnico, el sistema centmetro-gramo-segundo [cgs] y el sistema
metro-kilogramo-segundo [mks]. En la tabla 1.1 se resumen las distintas unidades de estos tres
sistemas.
Recordemos que la fuerza de atraccin gravitacional que la tierra ejerce sobre un cuerpo se
llama el peso del cuerpo. El peso, que es una fuerza, se expresa en unidades de fuerza. As, en el
sistema britnico, el peso se mide en libras; en el sistema [cgs], en dinas; y en el sistema [mks], en
newtons.
Aplicemos ahora la segunda ley a un cuerpo que cae libremente (un cuerpo que cae hacia la
tierra en ausencia de resistencia al aire). Que la masa del cuerpo sea m y que w denote su peso.
La nica fuerza que acta sobre el cuerpo es su peso y por lo tanto esta es la fuerza resultante. La
aceleracin es que debido a la gravedad, denotada por g, que es aproximadamente 32 [pie/seg2 ] en
el sistema britnico, 980 [cm/seg2 ] en el sistema [cgs], y 9.8 [m/seg2 ] en el sistema [mks] (para
puntos cerca de la superficie de la tierra). La segunda ley de Newton F = ma se reduce as a w = mg.
As
w
m= , (1.22)
g

una relacin que emplearemos con frecuencia.


Consideremos ahora un cuerpo B en movimiento rectilneo, es decir, en movimiento a lo largo
de una lnea recta L. En L elegimos un punto de referencia fijo como origen O, una direccin fija
como positiva y una unidad de distancia. Entonces la coordenada x de la posicin de B del origen O
nos dice la distancia o desplazamiento de B. (Ver figura 3.3) La velocidad instantnea de B es la
tasa de cambio de tiempo de x:

dx
v= ,
dt
y la aceleracin instantnea de B es la tasa de cambio de tiempo de v:

dv d 2 x
a= = 2.
dt dt
Tenga en cuenta que x, v y a son cantidades vectoriales. Todas las fuerzas, desplazamientos,
velocidades y aceleraciones en la direccin positiva de L son cantidades positivas; mientras que
aquellos en la direccin negativa son cantidades negativas.
Si ahora aplicamos la segunda ley de Newton F = ma al movimiento de B a lo largo de L,
observando que

dv dv dx dv
= =v ,
dt dx dv dx
14 Captulo 1. Aplicaciones

podemos expresar la ley en cualquiera de las tres formas siguientes:

dv
m = F, (1.23)
dt

d2x
m = F, (1.24)
dt 2

dv
mv = F, (1.25)
dx
donde F es la fuerza resultante que acta sobre el cuerpo. La forma de usar depende de la forma
en que se expresa F. Por ejemplo, si F es una funcin del tiempo t solamente y deseamos obtener
la velocidad v como una funcin de t, usaramos (1.23); mientras que si F se expresa como una
funcin del desplazamiento x y deseamos encontrar v como una funcin de x, emplearamos (1.25).

1.6.1 Movimiento de una partcula en un medio resistente


La segunda ley de Newton y el principio de DAlembert
La segunda ley de Newton: el producto de la masa de un objeto y su aceleracin es igual a la
suma de fuerzas aplicadas al objeto, es decir, ma = F.
Principio de DAlembert: reescribir la segunda ley de Newton como F ma = 0. Trata a ma
como una fuerza, conocida como la fuerza de inercia. Un objeto est en equilibrio (dinmico) bajo
la accin de todas las fuerzas aplicadas, incluida la fuerza de inercia. Esto se conoce como Principio
de DAlembert, que transforma un problema en dinmica en un problema de equilibrio esttico.

Principio de Impulso - Momento


Para un sistema de partculas, el cambio en el momento del sistema es igual al impulso total en
el sistema, es decir,

Momento en el tiempo t2 Momento en el tiempo t1 = Impulso durante t2 t1 .

El impulso de una masa m que se mueve a la velocidad v es igual a mv. El impulso de una fuerza F
durante el intervalo de tiempo Dt es igual a FDt.
Considere el movimiento de una partcula que se mueve en un medio resistente, como el aire o
el agua. El medio ejerce una fuerza de resistencia R sobre la partcula. En muchas aplicaciones,
la fuerza de resistencia R es proporcional a vn , donde v es la velocidad de la partcula y n > 0, y
es opuesta a la direccin de la velocidad. Por lo tanto, la fuerza de resistencia se puede expresar
como R = b vn , donde b es una constante. Para las partculas que se mueven en un medio viscoso
ilimitado a baja velocidad, la fuerza de resistencia es R = b v, es decir, n = 1.
A continuacin, se estudiar el caso con R = b v para el movimiento en la direccin vertical y
las condiciones iniciales especficas.

Caso I: Movimiento ascendente


Considere un objeto que se lanza verticalmente en el momento t = 0 desde x = 0 con la
velocidad inicial v0 como se muestra en la Figura 1.1.
El desplazamiento x, la velocidad v = x y la aceleracin a = v = x se toman como positivos en la
direccin ascendente. La partcula est sometida a dos fuerzas: la gravedad descendente mg y la
fuerza de resistencia del medio R = b v, que es opuesta a la direccin de la velocidad y, por lo tanto,
1.6 Problemas en Mecnica 15

es descendente.
De la segunda ley de Newton, la ecuacin del movimiento es

dv w
" ma = F : m = R mg, R = b v, m= ,
dt g

Figura 1.1: Movimiento ascendente de una partcula en un medio resistente.

dv b
= g(av + 1), a= > 0,
dt w

Z Z
dv 1
= g dt + c =) ln(av + 1) = gt + c. (1.26)
av + 1 a
la constante c se determina a partir de la condicin inicial t = 0, v = v0 :

1
ln(av0 + 1) = 0 + c
a
Sustituyendo en la ecuacin (1.26) se obtiene

1 1
ln(av + 1) = gt + ln(av0 + 1),
a a

1 av + 1 av + 1 agt
ln = gt =) =e .
a av0 + 1 av0 + 1
Resolver para v conduce a
agt
av = e (av0 + 1) 1. (1.27)

Cuando el objeto alcanza la altura mxima en el momento t = tmax , v = 0, y

agtmax 1
a 0 = e (av0 + 1) 1 =) tmax = ln(av0 + 1).
ag
dx
Para determinar el desplazamiento x(t), tenga en cuenta que v = dt y use la ecuacin (1.27)

dx 1 agt 1
= (av0 + 1)e .
dt a a
16 Captulo 1. Aplicaciones

La integracin con respecto a x da


Z
1 agt t 1 agt t
x= (av0 + 1) e dt +d = (av0 + 1)e + d.
a a a 2g a

La constante d se determina a partir de la condicin inicial t = 0, x = 0:

1 1
0= (av0 + 1)e0 0+d =) d= (av0 + 1).
a 2g a 2g

Por lo tanto
1 agt t
x= (av0 + 1)(e 1) .
a 2g a

En el momento t = tmax , el objeto alcanza la altura mxima dada por

1 ln(av0 +1) 1 1 1
x = xmax = (av0 + 1)[e 1] ln(av0 + 1) = 2 [av0 ln(av0 + 1)].
a 2g a ag a g

Caso II: Movimiento descendente


Considere la posibilidad de lanzar y soltar un objeto en el tiempo t = 0 de x = 0 con v = 0,
como se muestra en la Figura 1.2.

Figura 1.2: Movimiento hacia abajo de una partcula en un medio resistente.

En este caso, es ms conveniente tomar x, v y a como positivos en sentido descendente. La


Segunda Ley de Newton requiere

dv w
# ma = F : m = mg R, R = b v, m= ,
dt g

donde
dv b
=g agv, a= > 0,
dt w
La ecuacin se puede resolver fcilmente como
Z Z
dv 1
= g dt + c =) ln |1 av| = gt + c.
1 av a
donde la constante c se determina a partir de la condicin inicial t = 0, v = 0:

1
ln 1 = 0 + c =) c = 0,
a
1.6 Problemas en Mecnica 17

por lo tanto
1 1 agt
ln |1 av| = gt =) v= (1 e ). (1.28)
a a
Cuando el tiempo t se acerca al infinito, la velocidad se aproxima a una constante, la llamada
velocidad terminal,
1
v = vterminal = lm v =
t! a
El cambio de velocidad con el tiempo se muestra en la Figura 1.3.

Figura 1.3: Velocidad de una partcula que se mueve hacia abajo en un medio resistente.

Para encontrar el desplazamiento, vuelva a escribir la ecuacin (1.28) como


dx 1 agt
= (1 e ).
dt a
Integrando obtenemos
t 1 agt
x= + e + d,
a a 2g
donde la constante d est determinada por la condicin inicial t = 0, x = 0:
1 0 1
0 = 0+ e +d =) d= .
a 2g a 2g
Por lo tanto, el desplazamiento est dado por
t 1 agt
x= + 2 (e 1).
a a g
Ejemplo 1.6 Se dispara una bala perpendicularmente a una placa a una velocidad inicial de
v0 = 100 [m/seg]. Cuando la bala sale de la placa, su velocidad es v1 = 80 [m/seg]. Se sabe que el
espesor de la placa es b = 0,1 [m] y la fuerza de resistencia de la placa a la bala es proporcional al
cuadrado de la velocidad de la bala, es decir, R = b v2 . Determine el tiempo T que tarda la bala en
atravesar la placa.
Solucin Aplicando la Segunda Ley de Newton a la vieta como se muestra en la Figura 1.4,
entonces
dv
! ma = F : m = b v2 .
dt
18 Captulo 1. Aplicaciones

Figura 1.4:

La solucin general est dada por


Z Z
dv b 1 b
= dt + c =) = kt + c, k= ,
v2 m v m
donde la constante c se determina a partir de la condicin inicial t = 0, v = v0 :
1 1
= k0+c =) c= .
v0 v0
Por lo tanto
1 1 dx 1
= kt + =) v= = . (1.29)
v v0 dt kt + v10

La integracin con respecto a t conduce a



1 1
x = ln kt + + d,
k v0
donde la constante d se determina a partir de la condicin inicial t = 0, x = 0:

1 1 1 1
0 = ln k 0 + + d =) d = ln .
k v0 k v0
Por lo tanto

1 1 1 1
x= ln kt + ln , (1.30)
k v0 k v0
De la ecuacin (1.29),
1 1
t = T, v = v1 : = kT + . (1.31)
v1 v0
De la ecuacin (1.30), t = T , x = b:

1 1 1 1 1 v0 1 v0
b = ln kT + ln = ln =) k= ln ,
k v0 k v0 k v1 b v1
Usando la ecuacin (1.31),
1 1 1 1
1 1 1 v1 v0 80 100
T= =b = 0,1 0,000819 [seg].
k v1 v0 ln vv01 ln 100
80

1.6 Problemas en Mecnica 19

Ejemplo 1.7 Un objeto de masa m cae contra la resistencia del aire que es proporcional a la
velocidad (es decir, R = b v) y bajo gravedad g.
1. Si v0 y vE son las velocidades inicial y final (terminal), y v es la velocidad en el tiempo t,
muestre que
v vE kt b
=e , k= .
v0 vE m
2. La velocidad del objeto es de 30, 40, 45 [m/seg] en t = 1, 2 y 3 [seg], respectivamente, despus
de comenzar. Encuentre vE y v0 .
3. A qu hora la velocidad del objeto ser 49 [m/seg]?.
Solucin 1. El objeto est sujeto a dos fuerzas como se muestra: la gravedad hacia abajo mg y
la resistencia al aire hacia arriba b v, de acuerdo a la Figura 1.5.

Figura 1.5:

La Segunda Ley de Newton requiere


dv dv b
# ma = F : m = mg bv =) =g kv, k= .
dt dt m
Observando que g kv > 0, la ecuacin es variable separable y la solucin es
Z Z
dv 1 g kt
= dt + c =) ln (g kv) = t + c =) v= ce .
g kv k k
Constante c se determina a partir de la condicin inicial t = 0, v = v0 :
g g
v0 = ce0 =) c= v0 .
k k
Cuando t ! , v = vE , ve = gk , c = vE v0 . Por lo tanto, la velocidad viene dada por

kt v vE kt
v = vE (vE v0 )e =) =e . (1.32)
v0 vE
2. De la ecuacin (1.32),
30 vE
t = 1, v = 30 : = e k, (1.33)
v0 vE
40 vE
t = 2, v = 40 : = e 2k , (1.34)
v0 vE
45 vE
t = 3, v = 45 : = e 3k . (1.35)
v0 vE
Ya que e k e 3k = e 4k = (e 2k )2 ,

30 vE 45 vE 40 vE 2
= .
v0 vE v0 vE v0 vE
20 Captulo 1. Aplicaciones

(30 vE )(45 vE ) = (40 vE )2 =) vE = 50 [m/seg].

2
30 vE
v0 vE e 2k (30 VE )2 (30 50)2
= 2k
=1 =) v0 = + vE = + 50 = 10 [m/seg].
40 vE e 40 vE 40 50
v0 vE

3. De la ecuacin (1.33),

30 vE 30 50 1
k= ln = ln = ln = ln 2,
v0 vE 10 50 2

de donde

kt v vE 49 50 1 1 1 ln 40
e = = = =) t= ln = [seg].
v0 vE 10 50 40 k 40 ln 2

1.6.2 Cables colgantes

El puente colgante

Un puente colgante tpico consiste en cables, muelles (torres), anclajes, suspensiones (tirantes)
y plataforma (viga de refuerzo) como se muestra en la Figura 1.6. Normalmente, los pesos propios
de los cables son insignificantes en comparacin con la carga que transportan. La carga en los
cables proviene de la carga en la plataforma, que incluye el peso propio de la plataforma y la carga
de trfico, y es transmitida por los ganchos.

Figura 1.6: Puente colgante.

Considere un cable compatible con dos soportes A y B como se muestra en la Figura 1.7 (a). La
carga en el cable se modela como una carga distribuida w(x). Configure el sistema de coordenadas
cartesianas colocando el origen en el punto ms bajo del cable.
1.6 Problemas en Mecnica 21

Figura 1.7: Un cable bajo carga distribuida.

Para establecer la ecuacin diferencial gobernante, considere el equilibrio de un segmento de


cable entre 0 y x como se muestra en la Figura 1.7 (b). El cable est sujeto a tres fuerzas:
1. la fuerza de tensin horizontal H en el extremo izquierdo,
2. la fuerza de tensin T (x) tangente al cable en el extremo derecho, y
3. la porcin de la carga distribuida w(x) entre 0 y x. Puede ser reemplazado por su resultado
W (x) aplicado en el centroide del rea encerrada por la curva de intensidad de carga w(x) (el rea
sombreada).
El equilibrio del segmento requiere que

! Fx = 0 : T (x) cos q H = 0, (1.36)

Z x
" Fy = 0 : T (x) sin q W (x) = 0, w(x) dx. (1.37)
0

Eliminando T (x) de estas dos ecuaciones rinde

T (x) sin q W (x) W (x)


= =) tan q = .
T (x) cos q H H
De la geometra, uno tiene
dy dy W (x)
tan q = =) = .
dx dx H
Diferenciar con respecto a x lleva a

d2y 1 dW (x) w(x) dW (x)


2
= = , =) = w(x).
dx H dx H dx
22 Captulo 1. Aplicaciones

Supongamos que la carga est distribuida uniformemente, es decir, w(x) = w. La ecuacin diferen-
cial se convierte
d2y w
2
= . (1.38)
dx H
dy
Como el origen se toma en el punto ms bajo, uno tiene x = 0, y = 0, dx = 0.
Integrando la ecuacin (1.38) una vez, resulta

dy w
= x + c,
dx H
dy
donde la constante c se determina a partir de la condicin inicial x = 0, y = 0, dx = 0:

0 = 0+c =) c = 0,

La integracin nuevamente conduce a


w 2
y= x + d,
2H
donde la constante d se determina a partir de la condicin inicial x = 0, y = 0:

0 = 0+d =) d = 0,

Por lo tanto, la forma del cable es una parbola dada por


w 2
y= x . (1.39)
2H
Los huecos hA y hB se pueden determinar a partir de la ecuacin (1.39)

w 2 wLA2
cuando x = LA , y = hA : hA = L =) H= , (1.40)
2H A 2hA

w 2 wLB2
cuando x = LB , y = hB : hB = L =) H= . (1.41)
2H B 2hB
De las ecuaciones (1.40) y (1.41), uno obtiene la relacin entre LA , LB , hA y hB :

wLA2 wLB2 LA2 L2


H= = =) = B.
2hA 2hB hA hB
Para determinar la tensin en cualquier punto, use las ecuaciones (1.36) y (1.37)

T (x) cos q = H, T (x) sin q .

Al elevar al cuadrado ambos lados de estas dos ecuaciones y agregarlas, se genera

T 2 cos2 q + T 2 sin2 q = H 2 +W 2 (x) =) T 2 = H 2 +W 2 (x).

Dado que la carga est distribuida uniformemente, w(x) = w,


Z x Z x
W (x) = w(x) dx = w dx = wx.
0 0
1.6 Problemas en Mecnica 23

Por lo tanto, la tensin en cualquier punto est dada por


q p
T= H 2 +W 2 (x) = H 2 + w2 x2 . (1.42)

La tensin T es mxima cuando |x| es mximo. Por lo tanto, la tensin es mxima en el soporte
superior.

Figura 1.8: Un cable de puente colgante con soportes a la misma altura.

Para un cable de puente colgante con soportes a igual altura hA = hB = h, se tiene LA = LB = 12 L,


donde L es la longitud del tramo, como se muestra en la Figura 1.8. Luego, las ecuaciones (1.40) y
(1.41) dan la relacin entre el pandeo h y la tensin horizontal H en el punto ms bajo:

wL2 wL2
h= o H= . (1.43)
8H 8h

Ejemplo 1.8 Considere el cable principal de un puente colgante con una carga de intensidad
uniformemente distribuida w. Los dos soportes del cable estn a la misma altura. El tramo del cable
es L, el pandeo es h, y la rigidez axial es EA.
1. Derive una frmula para la elongacin d del cable.
2. Uno de los cables principales del tramo central del puente Golden Gate tiene las siguientes
propiedades: L = 1, 280 [m], h = 143 [m], w = 200 [kN/m], E = 200 [GPa]. El cable consta de
27.572 alambres paralelos de dimetro 5 [mm]. Determine el alargamiento de este cable.
Solucin 1. Considere un pequeo segmento de cable de longitud ds como se muestra en la
Figura 1.9. Est sujeto a las fuerzas de tensin axiales: T en el extremo izquierdo y T + dT en el
extremo derecho.

Figura 1.9:

Usando la frmula de elongacin de un miembro cargado axialmente en Mecnica de slidos

TL
d= ,
EA
24 Captulo 1. Aplicaciones

donde T es la fuerza axial, L es la longitud del miembro, E es el mdulo de Young, y A es el rea


de la seccin transversal, el alargamiento del segmento de cable ds es
T ds
dd = .
EA
Como la longitud del segmento del cable es
s 2 r
dy w 2 w 2
ds = 1 + = 1+ x dx, y= x
dx H 2H
y la tensin viene dada por la ecuacin (1.42)
p
T = H 2 + w2 x2 ,
se tiene
p r w 2 w 2
H 2 + w2 x2 H
dd = 1+ x dx = 1+ x dx
EA H EA H
La integracin en la longitud del tramo produce el alargamiento del cable
Z L w 2
2 H 2H L w2 L3
d =2 1+ x dx = +
0 EA H EA 2 3H 2 8
Usando la ecuacin (1.43) para expresar la tensin horizontal en trminos de la cada h, se obtiene
2 3
2 2 3

2 wL 6 L w L 7 wL3 16h2
d= 4 + 2 5= 1+ .
EA 8 2 wL2 8 8h EA 3L2
3 8h
2. El rea de la seccin transversal del cable es
p
A = 27,572 0,0052 0,5414 [m2 ]
8
La elongacin del cable es

wL3 16h2 200,000 1,2802 16 1432
d= 1+ = 1+ 3,61 [m].
8h EA 3L2 8 143 200 109 0,5414 3 1,2802

Cable bajo su propio peso


Hay muchas aplicaciones, como lneas de tela y cables de transmisin de energa, como se
muestra en la Figura 1.10, en la cual los cables estn suspendidos entre dos soportes bajo pesos
propios.

Figura 1.10: Cables de transmisin de energa.


1.6 Problemas en Mecnica 25

Considere un cable suspendido entre dos soportes como se muestra en la Figura 1.11 (a). El
cable se cuelga por su propio peso. Configure el sistema de coordenadas cartesianas colocando el
origen debajo del punto ms bajo del cable. La longitud s del cable se mide desde el punto ms
bajo.

Figura 1.11: Un cable por debajo del peso propio.

Para establecer la ecuacin diferencial gobernante, considere el equilibrio de un segmento del


cable de longitud s, como se muestra en la Figura 1.11 (b). El segmento de cable est sujeto a
tres fuerzas: el peso propio W (s), la fuerza de tensin horizontal H en el extremo izquierdo y la
fuerza de tensin T tangente al cable en el extremo derecho. El equilibrio de este segmento de cable
requiere
! Fx = 0 : T (x) cos q H = 0, (1.44)

" Fy = 0 : T (x) sin q W (s) = 0. (1.45)


Dividimos la ecuacin (1.45) para la ecuacin (1.44) obteniendo
T (x) sin q W (s) W (s)
= =) tan q = .
T (x) cos q H H
dy
Como tan q = dx , se obtiene
dy W (s)
= .
dx H
26 Captulo 1. Aplicaciones

o, despus de diferenciar la ecuacin con respecto a x,


d2y 1 dW (s)
= . (1.46)
dx2 H dx
Como la longitud de un segmento de cable es
s 2
dy
ds = 1 + dx.
dx
entonces, si el cable es uniforme,
dW (s)
= w = Densidad del peso del cable por unidad de longitud,
ds
y, usando la regla de la cadena,
s 2
dW (s) dW (s) ds dy
= =w 1+ ,
dx ds dx dx
lo que nos lleva a
s 2
d2y w dy
2
= 1+ , (1.47)
dx H dx
Esta es una ecuacin diferencial de segundo orden con x y y ausentes. Es ms fcil resolver la
dy du d2y
ecuacin como la del tipo y ausente. Hacemos u = dx , dx = dx 2 , la ecuacin (1.47) se convierte en

du w p
= 1 + u2 ,
dx H
Z Z w
du w 1 w
p = dx + c =) sinh u= x+c =) u = sinh x+c .
1+u 2 H H H
dy
La constante c se determina a partir de la condicin inicial x = 0, dx = 0:
w
0 = sinh 0+c =) c = 0.
H
Por lo tanto
w
u = sinh x .
H
Integrando conduce a
H w
y= cosh x + d.
w H
Cuando x = 0,
H H
y= cosh 0 + d = + d.
w w
Para simplificar la expresin de y, elija el origen tal que y = Hw cuando x = 0, lo que resulta en
d = 0.
Por lo tanto, la forma del cable como se muestra en la figura 1.12, que se llama catenaria, es decir
H w
y= cosh x . (1.48)
w H
1.6 Problemas en Mecnica 27

Figura 1.12: La forma del cable bajo el peso propio.

La curvatura del cable se puede determinar fcilmente en trminos de hA y hB



H H H w( LA )
En x = LA , y = + hA : + hA = cosh ,
w w w H

H H H wLB
En x = LB , y = + hB : + hB = cosh ,
w w w H


H wLA H wLB
hA = cosh 1 , hB = cosh 1 (1.49)
w H w H

La longitud del cable viene dada por

p = permetro
Z LB
ds
= dx
LA dx
s
Z LB
dy 2
= 1+ dx
LA dx
Z LB r w
= 1 + sinh2 x dx
LA H
H w LB
= sinh x
w H L
A
H wLB wLA
= sinh + sinh . (1.50)
w H H
Para determinar la tensin en cualquier punto, use las ecuaciones (1.44) y (1.45)
q
2 2 2 2 2 2
T cos q + T sin q = H +W (s) =) T = H 2 +W 2 (s),

donde s es la longitud del cable entre el punto de inters y el punto ms bajo, W (s) = ws es el peso
de este segmento de cable. Por lo tanto
q
T = H 2 +W 2 (s), (1.51)
28 Captulo 1. Aplicaciones

La tensin T es mxima cuando s es mxima. Por lo tanto, la tensin es mxima en el soporte


superior.
Para el caso especial cuando los dos soportes A y B estn a la misma altura, se tiene LA = LB = 12 L,
donde L es la longitud del tramo, hA = hB = h,

H wL
h= cosh 1 . (1.52)
w 2H

La longitud del cable es, a partir de la ecuacin (1.50),



2H wL
p= sinh . (1.53)
w 2H

Las tensiones en los dos soportes son las mismas y son mximas, dadas por
r
1 1
Tmx = H 2 + w2 p2 , smx = p. (1.54)
4 2
Ejemplo 1.9 Se suspende un cable de densidad de peso de 50 [N/m] en dos soportes de igual
altura. Los soportes estn separados por 10 [m] y la cada es de 2 [m]. Determine lo siguiente:
1. la tensin horizontal en el punto ms bajo;
2. la tensin en el soporte;
3. la longitud del cable.
Solucin Se conocen los siguientes parmetros: w = 50 [N/m], L = 10 [m], h = 2 [m]. De la
ecuacin (1.52),

H wL H 250
h= cosh 1 =) 2 = cosh 1 .
w 2H 50 2H

Esta es una ecuacin trascendental y se requiere un mtodo numrico para determinar su raz. Se
encuentra que H 327,93 [N].
Usando la ecuacin (1.53), la longitud del cable viene dada por

2H wL 2 327,93 50 10
p= sinh = sinh = 11,00 [m].
w 2H 50 2 327,93

La tensin en el soporte es, a partir de la ecuacin (1.54),


r r
1 1
Tmx = H 2 + w2 p2 = 327,932 + 502 11,002 = 427,98 [N].
4 4

Ejemplo 1.10 En la figura 1.13 (a) se muestra un flotador esfrico utilizado para marcar el
rumbo de una carrera de veleros. Una corriente de agua de izquierda a derecha provoca un arrastre
horizontal en el flotador. La longitud del cable entre los puntos A y B es de 60 [m], y la densidad
efectiva del cable es de 2 [kg/m] cuando se tiene en cuenta la flotabilidad del cable. Si se puede
descuidar el efecto de la corriente en el cable, determine las tensiones en los puntos A y B.
Solucin La densidad del peso del cable es w = 2 9,8 = 19,6 [N/m]. En este problema, el
punto ms bajo con pendiente cero no aparece entre los puntos A y B. Para aplicar la formulacin
establecida en esta seccin, agregue un segmento imaginario de cable CA como se muestra en la
Figura 1.13 (b), y coloque el origen debajo del punto ms bajo con pendiente cero una distancia de
H
w.
1.6 Problemas en Mecnica 29

Figura 1.13: Un flotador y un cable.

Aplicando la ecuacin (1.48) a los puntos A y B obtenidos, que denotan H0 = Hw ,


H w xA
Punto A : yA = cosh xA =) yA = H0 cosh ,
w H H0
H w xA + 30
Punto B : yB = cosh xB =) yA + 50 = H0 cosh .
w H H0
Restar estas dos ecuaciones conduce a

xA + 30 xA
50 = H0 cosh cosh . (1.55)
H0 H0

Siguiendo el mismo procedimiento que en la derivacin de la ecuacin (1.50), la longitud del cable
es
s 2
Z xB w xB
dy H xB xA
p= 1+ dx = sinh x = H0 sinh sinh .
xA dx w H xA H0 H0

Por lo tanto

xA + 30 xA
60 = H0 sinh sinh . (1.56)
H0 H0

Las ecuaciones (1.55) y (1.56) dan dos ecuaciones trascendentales para dos incgnitas H0 y xA .
Resolviendo las ecuaciones obtenemos
H
xA = 7,95 [m], H0 = = 19,14 =) H = H0 w = 19,14 19,6 = 375,14 [N].
w
La longitud de la curva CA es
s 2
Z xA w
dy H xA xA 7,95
SCA = 1+ dx = sinh x = H0 sinh = 19,14 sinh = 8,18 [m].
0 dx w H 0 H0 19,14

y la longitud de la curva CB es

SCB = SCA + sAB = 8,18 + 60 = 68,18 [m].


30 Captulo 1. Aplicaciones

Usando la ecuacin (1.51), las tensiones en los puntos A y B son


q q
TA = 2 =
H 2 + w2 sCA 375,142 + (19,6 8,18)2 = 408 [N],
q q
TB = 2 =
H 2 + w2 sCB 375,142 + (19,6 68,18)2 = 1,388 [N].

1.6.3 Circuitos elctricos


Hay tres elementos elctricos pasivos bsicos: resistencias, condensadores e inductores. La
resistencia R es la capacidad de los materiales para impedir el flujo de corriente, que se modela
mediante una resistencia.

Leyes bsicas
Ley de Ohm: v = iR, o i = Rv , donde v es el voltaje, i es la corriente.
Ley de corriente de Kirchhoff (KCL): La suma algebraica de todas las corrientes en
cualquier nodo en un circuito es igual a cero.
Ley de voltaje de Kirchhoff (KV L): La suma algebraica de todos los voltajes alrededor
de cualquier trayectoria cerrada en un circuito es igual a cero.
Un condensador es un componente elctrico que consta de dos conductores separados por un
aislante o material dielctrico. Si el voltaje vara con el tiempo, el campo elctrico vara con el
tiempo, lo que produce una corriente de desplazamiento en el espacio ocupado por el campo. La
capacitancia del parmetro del circuito C relaciona la corriente de desplazamiento con el voltaje
Z t Z t
dvC (t) 1 1
i(t) = C , vC (t) = i(t) dt = i(t) dt + vC (t0 ).
dt C C t0

R Un condensador se comporta como un circuito abierto en presencia de un voltaje constante.


El voltaje no puede cambiar abruptamente a travs de los terminales de un condensador.

Un inductor es un componente elctrico que se opone a cualquier cambio en la corriente


elctrica. Est compuesto por una bobina de alambre enrollada alrededor de un ncleo de soporte.
Si la corriente vara con el tiempo, el campo magntico vara con el tiempo, lo que induce un voltaje
en el conductor vinculado por el campo. La inductancia del parmetro del circuito L relaciona el
voltaje inducido con la corriente
Z t Z t
di(t) 1 1
vL (t) = L , i(t) = vL (t) dt = vL (t) dt + i(t0 ).
dt L L t0
1.6 Problemas en Mecnica 31

R Un inductor se comporta como un cortocircuito en presencia de una corriente constante. La


corriente no puede cambiar abruptamente en un inductor.

Cuatro tipos de circuitos simples (ver Figuras 3.11 y 3.12), un circuito que comprende una
resistencia y un condensador (circuito RC) y un circuito que comprende una resistencia y un
inductor (circuito RL), ya sea en serie o en paralelo, todos conducir a la ecuacin diferencial
ordinaria lineal de primer orden de la forma

dx 1
+ x = Q(t).
dt t

Figura 1.14: Circuitos RC.

La solucin viene dada por, con P(t) = t1 ,

R
Z R Z
t t s
P(t) dt P(t) dt
x(t) = e Q(t)e + c = ce t +e t Q(s)e t ds,

donde la constante c se puede determinar usando la condicin inicial: x(t) = x0 cuando t = 0,


entonces c = x0 . Por lo tanto, la solucin es
Z
t t s
x(t) = x0 e t +e t Q(s)e t ds.

Figura 1.15: Circuitos RL.

Si Q(t) = Q0 , la solucin se convierte en


t
x(t) = Q0 t + (x0 Q0 t)e t .
32 Captulo 1. Aplicaciones

Circuito Serie RC
Con referencia a la Figura 1.14 (a), aplicando lo obtenido de la Ley de Voltaje de Kirchhoff
Z t
1
V (t) + Ri + i(t) dt = 0.
C

Diferenciando con respecto a t da


di 1 dV (t) di 1 1 dV (t)
R + i= =) + i= ,
dt C dt dt RC R dt
1 dV (t)
en el cual x(t) = i(t), t = RC, Q(t) = R dt .

Circuito Paralelo RC
Con referencia a la Figura 1.14 (b), la aplicacin de la Ley actual de Kirchhoff en el nodo 1
resulta
v dv dv 1 I(t)
I(t) C =0 =) + v= ,
R dt dt RC C
I(t)
en el que x(t) = v(t), t = RC, Q(t) = C .

Circuito Serie RL
Con referencia a la Figura 1.15 (a), aplicando la Ley de Voltaje de Kirchhoff resulta
di di R V (t)
V (t) + Ri + L =0 =) + i= ,
dt dt L L

en el que x(t) = i(t), t = RL , Q(t) = V L(t) .


Circuito Paralelo RL
Con referencia a la Figura 1.15 (b), aplicando la Ley de Kirchhoff en el nodo 1 resulta
Z t
v 1
I(t) v(t) dt = 0.
R L

Diferenciando con respecto a t da


1 dv 1 dI(t) dv R dI(t)
+ v= =) + v=R ,
R dt L dt dt L dt
dI(t)
en el que x(t) = v(t), t = RL , Q(t) = R dt .

Ejemplo 1.11 Para el circuito elctrico que se muestra en la siguiente figura, determine vL para
t > 0.
1.6 Problemas en Mecnica 33

Solucin Para t < 0, el interruptor est cerrado y el inductor se comporta como un cortocir-
cuito.

Las tres resistencias de 12 [W], 20 [W] y 6 [W] estn en conexin en paralelo y se pueden
combinar como una resistencia equivalente
1 1 1 1 10
= + + =) Req = .
Req 12 20 6 3
Aplicando la Ley de la corriente de Kirchhoff en el nodo 1 obtenemos
v(0 ) v(0 ) 25 3v(0 ) v(0 )
4 = i(0 ) + i2 (0 ) = + = + 5,
Req 5 10 5

v(0 )
v(0 ) = 18 [V] =) iL (0 ) = = 3 [A] .
6
Para t = 0, el interruptor est abierto. Como la corriente en un inductor no puede cambiar abrupta-
mente, iL (0 ) = iL (0+ ) = 3 [A].
Para t > 0, el interruptor est abierto y el circuito se convierte

Es fcil evaluar eso


v vL
iL = =) v = 6iL + vL ,
6
v 6iL + vL v 25 6iL + vL 25
i1 = = , i2 = = .
20 20 5 5
Aplicando la Ley de la corriente de Kirchhoff en el nodo 1 rinde
6iL + vL 6iL + vL 25
i1 + i2 + iL = 0 =) + + iL = 0,
20 5
34 Captulo 1. Aplicaciones
diL
vL + 10iL = 20 + 10iL = 20.
=)
dt
1
Con t = 10 , Q0 = 20, iL (0+ ) = 3, la solucin de la ecuacin diferencial es
t
10t
iL (t) = Q0 t + [iL (0+ ) Q0 t]e t = 2+e ,
diL (t) 10t
vL (t) = 1 = 10e [V].
dt

Ejemplo 1.12 Para el circuito elctrico que se muestra en la siguiente figura, determine i1 por
t > 0.

Solucin Para t < 0, el interruptor est abierto y la fuente actual est desconectada. El
condensador se comporta como un circuito abierto. Por lo tanto, i1 (0 ) = 0 y v2 (0 ) = 0.

Para t = 0, el interruptor est cerrado. Como el voltaje del capacitor no puede cambiar abrupta-
mente, v2 (0+ ) = v2 (0 ) = 0. Aplicando la Ley de la corriente de Kirchhoff en el nodo 1 obtenemos
v1 (0+ ) v1 (0+ ) v2 (0+ ) v1 (0+ ) v1 (0+ )
I0 = i1 (0+ ) + i2 (0+ ) = + = + ,
R1 R2 R1 R2
R1 R2 v1 (0+ ) R2
v1 (0+ ) = I0 =) i1 (0+ ) = = I0 .
R1 + R2 R1 R1 + R2
Para t > 0, i1 = Rv11 entonces v1 = R1 i1 ,
v1 v2 R1 i1 v2 v2 dv2
i2 = = , i3 = , iC = C .
R2 R2 R3 dt
La aplicacin de la Ley de la corriente de Kirchhoff en el nodo 1 da
R1 i1 v2
I0 = i1 + i2 =) i2 = I0 i1 = =) v2 = (R1 + R2 )i1 R2 I0 .
R2
La aplicacin de la Ley de la corriente de Kirchhoff en el nodo 2 conduce a
dv2 v2
i2 = iC + i3 =) I0 i1 = C + ,
dt R3
1.6 Problemas en Mecnica 35

di1 (R1 + R2 )i1 R2 I0


I0 i1 = C(R1 + R2 ) + ,
dt R3
di1 1 C(R1 + R2 )R3 R2 + R3
+ i1 = Q0 , t= , Q0 = I0 .
dt t R1 + R2 + R3 C(R1 + R2 )R3
La solucin est dada por

t R2 + R3 R2 R2 + R3 t
i1 (t) = Q0 t + [i1 (0 )+
Q0 t]e t = I0 + I0 e t .
R1 + R2 + R3 R1 + R2 R1 + R2 + R3

R Como estos circuitos se caracterizan por ecuaciones diferenciales de primer orden, se deno-
minan circuitos de primer orden. Consisten en resistencias y el equivalente de un elemento de
almacenamiento de energa, como condensadores e inductores.

1.6.4 Purificacin natural en una corriente


En esta seccin, se investiga la variacin de la calidad del agua en una corriente debido a la
contaminacin. La cantidad de contaminante se considera lo suficientemente pequea como para
que el flujo de la corriente no se vea alterado por su presencia.
Cuando las aguas residuales y los desechos se descargan en una corriente, el agua de la
corriente se degradar en su calidad fsica (por ejemplo, olor y color), su contenido qumico y
el tipo y la poblacin de la vida acutica. El criterio para determinar la calidad del agua de la
corriente depende de los usos que el agua tiene para servir. Para muchos propsitos, los ingenieros
usan la concentracin de Oxgeno Disuelto (DO) y la materia orgnica descomponible en el
agua como indicadores de su calidad. La DO mide la capacidad del agua para asimilar muchos
materiales contaminantes y para apoyar la vida acutica. La materia orgnica consume oxgeno
en su descomposicin. En aguas residuales, la materia orgnica incluye una gran variedad de
compuestos, representados por la cantidad de oxgeno requerida para su descomposicin biolgica
(Demanda bioqumica de oxgeno o DBO).
El agua limpia de la corriente generalmente est saturada con DO. A medida que se agregan las
aguas residuales y fluyen en la corriente, el OD en el agua contaminada se consume a medida que
se descompone la materia orgnica. Mientras tanto, el oxgeno de la atmsfera se disuelve en el
agua, ya que ahora ya no est saturado con DO. Finalmente, la materia orgnica se descompone
por completo y el agua de la corriente se satura nuevamente con DO. Este proceso natural de
purificacin se lleva a cabo dentro de un perodo de varios das. Es necesario determinar la variacin
de DO y BOD a lo largo del flujo para determinar el efecto de la contaminacin en la corriente.
Se supone que la DBO aadida a la corriente se propaga a travs de la corriente en una distancia
que es muy corta en comparacin con la longitud de la corriente donde tiene lugar la desoxigenacin
por la DBO y la reoxigenacin por la atmsfera, por lo que el problema puede considerarse una
dimensin con DO y DBO se supone que es uniforme en una seccin transversal.
Para derivar las ecuaciones que rigen, considere el balance de masa de DBO durante dt en
un volumen A dx delimitado por dos secciones transversales dx separadas, como se muestra en la
Figura 3.13, donde A es el rea de la seccin transversal del flujo, x es la distancia medido a lo
largo de la corriente, y t es el tiempo.
Emplee las siguientes anotaciones:
Q = la descarga,
b = la concentracin de DBO en masa por unidad de volumen de agua,
c = la concentracin de DO en masa por unidad de volumen de agua,
M = la masa de DBO aadida por unidad de tiempo por descarga unitaria a lo largo del flujo,
36 Captulo 1. Aplicaciones

N = la masa de oxgeno agregado por unidad de tiempo por unidad de descarga a lo largo de la
corriente procedente de fuentes distintas de la atmsfera (por ejemplo, a partir de la fotosntesis de
plantas verdes en la corriente),
r1 = la masa de DBO descompuesta por unidad de volumen por unidad de tiempo,
r2 = la tasa de reoxigenacin atmosfrica en masa por unidad de volumen por unidad de tiempo.

Figura 1.16: Purificacin natural en una corriente.

Cualquier diferencia entre entrada y salida y entre suma y resta causar un cambio en la masa
de DBO contenida entre estas dos secciones transversales. Por lo tanto, durante dt,

Q b (bA)
Q b dt Q+ dx b+ dx dt + M dx dt r1 A dx dt = dx dt.
x x t

Simplificando esta ecuacin dividiendo para (A dx dt) obtenemos

b b Q b M
+V = r1 + , (1.57)
t x A A
Q
donde V = A es la velocidad media en una seccin transversal, y

Q A
Q = + ,
x t
es la descarga agregada por unidad de longitud de flujo. Del mismo modo, a partir de un balance de
masa para el DO, se tiene

Q c (cA)
Q c dt Q+ dx c+ dx dt r1 A dx dt + r2 A dx dt + N dx dt = dx dt.
x x t

Simplificar la ecuacin dividiendo (A dx dt) lleva a

c c Q c N
+V = r1 + r2 + , (1.58)
t x A A
En general, la tasa r1 de consumo de oxgeno depende, entre otros factores, de la concentracin
de DO c y de la concentracin de DBO b. En casos prcticos, la concentracin de DBO suele ser
suficientemente baja, de modo que se puede suponer que r1 es proporcional a la concentracin
de DBO e independiente de la concentracin de DO, siempre que sea mayor que un valor muy
pequeo; es decir

r1 = k1 b, para c > 0+ , (1.59)


1.6 Problemas en Mecnica 37

donde el coeficiente k1 depende de la composicin del alcantarillado y su temperatura. Para un


alcantarillado dado, el valor numrico de k1 se puede determinar en el laboratorio.
La tasa de r2 de reoxigenacin atmosfrica generalmente se supone que es proporcional al
dficit de DO (cs c):

r2 = k2 (cs c), para 0 c cs , (1.60)

donde cs es la concentracin de saturacin de DO, que depende de la temperatura del agua. El


coeficiente k2 depende de la temperatura, el rea de la interfaz aire-agua por unidad de volumen de
la corriente y la turbulencia del aire y el agua.
Con los datos hidrogrficos y las fuentes de DBO y DO del flujo, la ecuacin (1.57) se puede
resolver de forma independiente para la distribucin de DBO b(x, t). La distribucin de DO c(x, t)
se puede obtener de la ecuacin (2), ya que r1 = k1 b ahora es una funcin conocida de x y t.
Para simplificar el anlisis, se considera el caso de estado estacionario, en el que las variables,
como b(x, t) y c(x, t), no cambian con el tiempo t. Por lo tanto, las ecuaciones (1.57) y (1.58) se
vuelven

db 1 dQ k1 M
+ + b= , (1.61)
dx Q dx V Q

dc 1 dQ k1 k2 N
+ c+ b (cs c) = ,
dx Q dx V V Q
que puede ser reescrito como

d(cs c) 1 dQ k2 k1 1 dQ N
+ + (cs c) = b+ cs + , (1.62)
dx Q dx V V Q dx Q

A continuacin, se estudian varios casos especiales.


1. Considera b(x) como la concentracin de BOD en una corriente constante de Q y V . Determine
b(x) para el caso con b = b0 a x = 0 y M = 0. Determine la distribucin constante de DO a lo largo
de la ruta con c = c0 a x = 0 y N = 0.
Como M = 0, Q y V son constantes, dQ dx = 0, la ecuacin (1.61) se convierte en

db k1
+ b = 0,
dx V
la solucin de que es
Z Z
db k1 k1
= dx + c =) ln b = x + ln d.
b V V
Por lo tanto
k1 x
b(x) = de V ,

donde la constante d est determinada por la condicin inicial b = b0 a x = 0:


k1 x
b0 = de0 =) d = b0 =) b(x) = b0 e V .

Para N = 0, la ecuacin (1.62) se convierte

d(cs c) k2 k1 k1 x
+ (cs c) = b0 e V ,
dx V V
38 Captulo 1. Aplicaciones

en el que la variable dependiente es (cs c). Es fcil de evaluar

k2 k1 k1 x
P(x) = , Q(x) = b0 e V ,
V V
Z
k2 R
P(x) dx
k2 x R
P(x) dx
k2 x
P(x) dx = x, e =e V , e =e V ,
V
Z Z
R
P(x) dx k1 k1 x k2 x k1 b0 (k2 k1 )x
Q(x)e dx = b0 e V e V dx = e V ,
V k2 k1
La solucin general de la ecuacin diferencial es
Z
R
P(x) dx
R k2 x k1 b0 (k2 k1 )x
(cs c) = e Q(x)e P(x) dx dx + c = e V e V +c ,
k2 k1

donde la constante c est determinada por la condicin inicial c = c0 a x = 0:

k1 b0 k1 b0
(cs c0 ) = +c =) c = (cs c0 ) .
k2 k1 k2 k1
Por lo tanto
k2 x k1 b0 k1 x k2 x

cs c(x) = (cs c0 )e V + e V e V .
k2 k1

2. Determine la distribucin BOD b(x) para el caso con Q = Q0 (1 + gx), constante V , M y k1 , y


b = b0 a x = 0. Determine la distribucin de OD en la ruta con una constante N y c = c0 a x = 0.
De la solucin, encuentre el valor de c muy ro abajo para el caso de un flujo uniforme (g = 0).
Como V , M y k1 son constantes, Q = Q0 (1 + gx) entonces dQ dx = Q0 g, la ecuacin (1.61) se
convierte en

db g k1 M
+ + b= ,
dx 1 + gx V Q0 (1 + gx)
db
que es de la forma dx + P(x)b = Q(x), donde

g k1 M
P(x) = + , Q(x) = ,
1 + gx V Q0 (1 + gx)
Z Z
g k1 k1
P(x) dx = + dx = ln (1 + gx) + ,
1 + gx V V
R
P(x) dx
k1 x k1 x R 1 k1 x
e = eln (1+gx) e V = ln (1 + gx) e V , e P(x) dx
= e V ,
ln 1 + gx
Z Z
R
P(x) dx M k1 x MV k1 x
Q(x) e dx = (1 + gx) e V dx = eV ,
Q0 (1 + gx) Q 0 k1
La solucin general de la ecuacin diferencial es
Z R
b(x) = e intP(x) dx Q(x)e P(x) dx dx + c

1 k1 x MV k1 x
= e V e V +c
1 + gx Q0 k1
MV 1 c k1 x
= + e V ,
Q0 k1 1 + gx 1 + gx
1.6 Problemas en Mecnica 39

en el que la constante c est determinada por la condicin inicial b = b0 a x = 0:


MV MV
b0 = + c =) c = b0 .
Q0 k1 Q0 k1
Por lo tanto

1 MV MV k1 x MV 1 k1 x
b0 k1 x
b(x) = + b0 e V = 1 e V + e V ,
1 + gx Q0 k1 Q0 k1 Q0 k1 1 + gx 1 + gx
La ecuacin (1.62) se convierte

d(cs c) g k2 k1 MV 1 k1 x
b0 k1 x
+ + (cs c) = 1 e V + e V +
dx 1 + gx V V Q0 k1 1 + gx 1 + gx
gcs N
+ + ,
1 + gx Q0 (1 + gx)
que es una ecuacin diferencial lineal de primer orden con una variable dependiente que es (cs c),
y
g k2 a b k1 x
P(x) = + , Q(x) = + e V ,
1 + gx V 1 + gx 1 + gx
dnde
k1 MV N M+N k1 b0 M
a= + gcs + = gcs + , b= .
V Q 0 k1 Q0 Q0 V Q0
En referencia a la ecuacin diferencial para b(x), se tiene
R
P(x) dx
k2 x R
P(x) dx 1 k2 x
e = (1 + gx)e V , e = e V ,
1 + gx
Z Z
R
P(x) dx a b k1 x k2 x
Q(x) e dx = + e V (1 + gx) e V dx
1 + gx 1 + gx
Z h k2 x (k2 k1 )x
i
= a e V +b e V dx
aV k2 x bV (k2 k1 )x
eV +
= e V ,
k2 k2 k1
La solucin general de la ecuacin diferencial es
R
Z R
P(x) dx
(cs c) = e Q(x) e P(x) dx dx + c

1 k2 x aV k2 x bV (k2 k1 )x
= e V eV + e V +c ,
1 + gx k2 k2 k1
donde la constante c est determinada por la condicin inicial c = c0 a x = 0:
aV bV aV bV
(cs c0 ) = + +c =) c = (cs c0 ) .
k2 k2 k1 k2 k2 k1
Por lo tanto

1 k2 x aV k2 x bV (k2 k1 )x
cs c(x) = e V (cs c0 ) + eV 1 + e V 1 .
1 + gx k2 k2 k1
Para un flujo uniforme, g = 0, y para muy abajo, x ! . Al tomar el lmite como x ! , obtenemos,

k2 x aV k2 x
bV k1 x k2 x
aV
lm [cs c(x)] = lm (cs c0 )e V + 1 e V + e V e V = ,
x! x! k2 k2 k1 k2
entonces
aV
lm c(x) = cs .
x! k2
40 Captulo 1. Aplicaciones

Procedimiento para resolver un problema de aplicacin


1. Establezca las ecuaciones diferenciales gobernantes basadas en los principios fsicos y las
propiedades geomtricas que subyacen al problema.
2. Identifica el tipo de estas ecuaciones diferenciales y luego resulvelas.
3. Determina las constantes arbitrarias en las soluciones generales usando las condiciones
iniciales o de contorno.
Ejemplo 1.13 Un ferry cruza un ro de ancho a desde el punto A hasta el punto O como se
muestra en la siguiente figura. El bote siempre apunta hacia el destino O. La velocidad del flujo del
ro es constante vR y la velocidad del bote es constante vB . Determine la ecuacin de la ruta trazada
por el bote.

Solucin Supongamos que, en el momento t, el barco est en el punto P con las coordenadas
(x, y). La velocidad del barco tiene dos componentes: la velocidad del barco vB con relacin al
flujo del ro (como si el ro no fluye), que apunta hacia el origen O o a lo largo de la lnea PO, y el
velocidad del ro vR en la direccin y.
Descomponiendo los componentes de velocidad vB y vR en las direcciones x y y
vx = vB cos q , vy = vR vB sin q .
Para DOHP, es fcil de ver
OH x PH y
cos q = =p , sin q = =p .
OP x2 + y2 OP x 2 + y2
Por lo tanto, las ecuaciones de movimiento estn dadas por
dx x dy y
vx = = vB p , vy = = vR vB p .
dt x + y2
2 dt x + y2
2

Como solo se busca la ecuacin entre x y y, la variable t puede eliminarse dividiendo estas dos
ecuaciones
dy vR vB p 2y 2 p r y 2 y
dy dt x +y k x 2 + y2 y vR
= dx = x = = k 1 + + , k= .
dx dt
vB p 2 2 x x x vB
x +y

y dy du
Sea u = x o y = xu, dx = u+x dx . Por lo tanto, la ecuacin se convierte
du p du p
u+x = k 1 + u2 + u =) x = k 1 + u2
dx dx
La solucin general es
Z Z p p
du dx
p = k +d =) ln (u + 1 + u2 ) = k ln x+ln c =) u+ 1 + u2 = cx k .
1 + u2 x
1.6 Problemas en Mecnica 41

Reemplazando u por los rendimientos de las variables originales


r y 2
y p
+ 1+ = cx k =) x2 + y2 = cx1 k y.
x x
La cuadratura de ambos lados conduce a

x2 + y2 = c2 x2(1 k)
2cx1 k y + y2 =) x2 = c2 x2(1 k)
2cx1 k y.

La constante c est determinada por la condicin inicial t = 0, x = a, y = 0:

a2 = c2 a2(1 k)
0 =) c = ak .

Por lo tanto, la ecuacin de la ruta es



1 2 2(1 k) 1 a x 1 k x 1+k
y= [c x x ] = (ak x1
2 k k 1+k
a x ) =) y= .
2cx1 k 2 2 a a

1.6.5 Problemas de cada de un cuerpo


Ahora consideraremos algunos ejemplos de un cuerpo que cae a travs del aire hacia la
tierra. En tal circunstancia el cuerpo encuentra resistencia del aire cuando cae. La cantidad de
resistencia del aire depende de la velocidad del cuerpo, pero no se conoce ninguna ley general
que exprese exactamente esta dependencia. En algunos casos, la ley R = kv parece ser bastante
satisfactoria, mientras que en otros R = kv2 parece ser ms exacta. En cualquier caso, la constante
de proporcionalidad k a su vez depende de varias circunstancias. En los ejemplos que siguen,
asumiremos ciertas leyes de resistencia razonables en cada caso. De este modo, trataremos realmente
de problemas idealizados en los que se aproxima la verdadera ley de resistencia y en la que se
desprecian ciertos factores comparativamente insignificantes.
Ejemplo 1.14 Un cuerpo que pesa 8 [lb] cae del resto hacia la tierra desde una gran altura.
Cuando cae, la resistencia del aire acta sobre ella, y asumiremos que esta resistencia (en libras) es
numricamente igual a 2v, donde v es la velocidad (en pies por segundo). Encuentra la velocidad y
la distancia cadas en el tiempo t segundos.
Solucin Elegimos el eje x positivo verticalmente hacia abajo a lo largo de la trayectoria del
cuerpo B y el origen en el punto desde el cual cay el cuerpo. Las fuerzas que actan sobre el
cuerpo son:
F1 , su peso, 8 [lb], que acta hacia abajo y por lo tanto es positivo.
F2 , la resistencia del aire, numricamente igual a 2v, que acta hacia arriba y por lo tanto es la
cantidad negativa 2v.
Vase la figura 3.4, donde se indican estas fuerzas. La segunda ley de Newton, F = ma, se convierte
en
dv
m = F1 + F2
dt
w 8
o, tomando g = 32 y usando m = g = 32 = 14 ,

1 dv
=8 2v. (1.63)
4 dt
Dado que el cuerpo estaba inicialmente en reposo, tenemos la condicin inicial

v(0) = 0. (1.64)
42 Captulo 1. Aplicaciones

La ecuacin (1.63) es separable. separando variables, tenemos

dv
= 4 dt.
8 2v
Integrando encontramos

1
ln |8 2v| = 4t + c0 ,
2
que se reduce a
8t
8 2v = c1 e .

Aplicando la condicin (1.64) encontramos c1 = 8. As, la velocidad en el tiempo t es dada por


8t
v = 4(1 e ). (1.65)

Ahora para determinar la distancia cada en el tiempo t, escribimos (1.65) en la forma

dx 8t
= 4(1 e )
dt
y tenga en cuenta que x(0) = 0. Integrando las ecuaciones anteriores, obtenemos

1 8t
x=4 t+ e + c2
8
1
Dado que x = 0 cuando t = 0, encontramos c2 = 2 y por lo tanto la distancia cada es dada por

1 8t 1
x=4 t+ e . (1.66)
8 8

Ejemplo 1.15 Un paracaidista equipado con paracadas y otro equipo esencial cae del descanso
hacia la tierra. El peso total del hombre ms el equipo es de 160 [lb]. Antes de que se abra el
paracadas, la resistencia del aire (en libras) es numricamente igual a 12 v, donde v es la velocidad
(en pies por segundo). el paracadas se abre 5 [seg] despus de que comience la cada; despus de
que se abre, la resistencia del aire (en libras) es numricamente igual a 58 v2 , donde v es la velocidad
(en pies por segundo). Encontrar la velocidad del paracaidista (a) antes de que se abra el paracadas,
y (b) despus de que se abra el paracadas.
Solucin De nuevo escogemos el eje positivo x verticalmente hacia abajo con el origen en el
punto donde comenz la cada. El enunciado del problema sugiere que lo dividimos en dos partes:
(a) antes de que se abra el paracadas; (b) despus de abrir.
Consideremos primero el problema (a). Antes de que se abra el paracadas, las fuerzas que
actan sobre el paracaidista son:
1. F1 , el peso, 160 [lb], que acta hacia abajo y por lo tanto es positivo.
2. F2 , la resistencia del aire, numricamente igual a 12 v, que acta hacia arriba y por lo tanto es la
cantidad negativa 12 v.
Usamos la segunda ley de Newton F = ma, donde F = F1 + F2 , vamos m = wg y tomamos
g = 32. Obtenemos

dv 1
5 = 160 v.
dt 2
1.6 Problemas en Mecnica 43

Dado que el paracaidista estaba inicialmente en reposo, v = 0 cuando t = 0. as, el problema (a),
referido al tiempo antes de que se abra el paracadas, se formula de la siguiente manera:

dv 1
5 = 160 v. (1.67)
dt 2

v(0) = 0. (1.68)

Pasemos ahora a la formulacin del problema (b). Razonando como antes, vemos que despus de
que se abra el tobogn para, las fuerzas que actan sobre el paracaidista son:
1. F1 = 160, exactamente como antes.
2. F2 = 58 v2 (en lugar de 12 v).
As, procediendo como se ha indicado anteriormente, se obtiene la ecuacin diferencial

dv 5 2
5 = 160 v . (1.69)
dt 8
Dado que el paracadas se abre 5 [seg] despus de que comience la cada, tenemos v = v1 cuando
t = 5, donde v1 es la velocidad alcanzada cuando se abre el paracadas. Por lo tanto, el problema
(b), relacionado con el tiempo despus de que el paracadas se abre, se formula como sigue:

dv 5 2
5 = 160 v . (1.70)
dt 8

v(5) = v1 . (1.71)

Primero consideraremos el problema (a). Encontramos una familia de un solo parmetro de solucin
de (1.67). Separando variables, obtenemos

dv 1
= dt.
v 320 10
integrando, obtenemos

1
ln |v 320| = t + c0 ,
10
que simplifica fcilmente a la forma
1
t
v = 320 + ce 10 .

Aplicando la condicin inicial (1.68) que v = 0 a t = 0, encontramos que c = 320. Por lo


tanto, la solucin al problema (a) es
1
t
v = 320(1 e 10 ), (1.72)

que es vlido para 0 t 5. En particular, donde t = 5, obtenemos


1
v = 320(1 e 2 ) 126, (1.73)

que es la velocidad cuando se abre el paracadas.


44 Captulo 1. Aplicaciones

Ahora consideremos el problema (b). Primero encontramos una familia de soluciones de un


parmetro de la ecuacin diferencial (1.70). Simplificando separando variables, obtenemos
dv dt
= .
v2 256 8
Resultando de la integracin

1 v 16 t
ln = + c2
32 v + 16 8
o
v 16
ln = 4t + c1 .
v + 16
Esto simplifica fcilmente a la forma
v 16 4t
= ce , (1.74)
v + 16
y resolviendo esto por v obtenemos

16(ce 4t + 1)
v= . (1.75)
1 ce 4t
Aplicando la condicin inicial (1.71) que v = v1 a t = 5, donde v1 est dada por (1.73) y es
aproximadamente 126, a (1.74), obtenemos
110 20
c= e .
142
Sustituyendo esto en (1.75) obtenemos
110 20 4t
16 142 e +1
v= 110 20 4t
. (1.76)
1 142 e

que es vlido para t 5.

1.6.6 Fuerzas de friccin


Si un cuerpo se mueve sobre una superficie rugosa, no slo encontrar resistencia al aire sino
tambin otra fuerza de resistencia debido a la rugosidad de la superficie. Esta fuerza adicional se
llama friccin. Se demuestra en la fsica que la friccin est dada por N, donde
1. es una constante de proporcionalidad llamada coeficiente de friccin, que depende de la
rugosidad de la superficie dada; y
2. N es la fuerza normal (es decir, perpendicular) que la superficie ejerce sobre el cuerpo.
Ahora aplicamos la segunda ley de Newton a un problema en el que la friccin est involucrada.
Ejemplo 1.16 Un objeto que pesa 48 [lb] es liberado desde el reposo en la parte superior de un
plano de deslizamiento de metal que est inclinado 30 a la horizontal. La resistencia del aire (en
libras) es numricamente igual a la mitad de la velocidad (en pies por segundo) y el coeficiente de
friccin es de un cuarto.
a. Cul es la velocidad del objeto 2 [seg] despus de su liberacin?
b. Si la diapositiva tiene 24 [pies] de largo, cul es la velocidad cuando el objeto alcanza el
fondo?
Solucin La lnea de movimiento est a lo largo de la diapositiva. Elegimos el origen en la parte
1.6 Problemas en Mecnica 45

superior y la direccin positiva x abajo en la diapositiva. Si descuidamos temporalmente la friccin


y la resistencia del aire, las fuerzas que actan sobre el objeto A son:
1. Su peso, 48 [lb], que acta verticalmente hacia abajo; y
2. La fuerza normal, N, ejercida por la corredera que acta en una direccin ascendente perpendi-
cular a la corredera.
Los componentes del peso paralelo y perpendicular a la corredera tienen magnitud

48 sin 30 = 24

y
p
48 cos 30 = 24 3,

respectivamente. Los componentes perpendiculares


p a la corredera estn en equilibrio y por lo tanto
la fuerza normal N tiene magnitud 24 3.
Ahora, teniendo en cuenta la friccin y la resistencia del aire, vemos que las fuerzas que actan
sobre el objeto a medida que se mueve a lo largo de la diapositiva son las siguientes:
1. F1 , la componente del peso paralelo al plano, que tiene un valor numrico 24. Dado que esta
fuerza acta en la direccin positiva (hacia abajo) a lo largo de la corredera,
p tenemos F1 = 24.
1
2. F2 , la fuerza de friccin, que tiene el valor numrico N = 4 (24 3). Ya que esto acta en la
p
direccin negativa (hacia arriba) a lo largo de la corredera, tenemos F2 = 6 3.
3. F3 , la resistencia del aire, que tiene el valor numrico 12 v. Dado que v > 0 y esto tambin acta
en la direccin negativa, tenemos F3 = 12 v.
p
Aplicamos la segunda ley de Newton F = ma. Aqu F = F1 + F2 + F3 = 24 6 3 12 v y
m = wg = 48 3
32 = 2 . As tenemos la ecuacin diferencial

3 dv p 1
= 24 6 3 v. (1.77)
2 dt 2
Dado que el objeto se libera del reposo, la condicin inicial es

v(0) = 0. (1.78)

La ecuacin (1.77) es separable; separando las variables que tenemos

dv dt
p = .
48 12 3 v 3

Integrando y simplificando, encontramos


p 1
v = 48 12 3 c1 e 3 t .
p
La condicin (1.78) da c1 = 48 12 3. As obtenemos
p 1
v = (48 12 3)(1 e 3 t ). (1.79)

La pregunta (a) se responde as haciendo t = 2 en la ecuacin (1.79). Encontramos


p 2
v(2) = (48 12 3)(1 e 3 ) 13,2 [pie/seg].

Para responder a la pregunta (b), integramos (1.79) para obtener


p 1
x = (48 12 3)(t + 3e 3 t ) + c2 .
46 Captulo 1. Aplicaciones
p
Dado que x(0) = 0, c2 = (48 12 3)(3). As, la distancia cubierta en el tiempo t es dada por
p 1
t
x = (48 12 3)(t + 3e 3 3).

Dado que la diapositiva es de 24 [pies] de largo, el objeto alcanza el fondo en el momento T


determinado a partir de la ecuacin trascendental
p 1
T
24 = (48 12 3)(T + 3e 3 3).

que puede escribirse como


p
1
T 47 + 2 3
3e 3 = T.
13
El valor de T que satisface esta ecuacin es aproximadamente 2.6. As, a partir de la ecuacin
(1.79), la velocidad del objeto cuando alcanza el fondo se da aproximadamente
p
(48 12 3)(1 e 0,9 ) 16,2 [pie/seg].

1.7 Ejercicios
1. Una piedra de 4 [lb] cae de reposo hacia la tierra desde una gran altura. Cuando cae, acta
1
sobre la resistencia del aire que es numricamente igual a v [lb], donde v [pies/seg] es la
2
velocidad.
a) Encuentre la velocidad y la distancia al tiempo cado t [seg].
b) Encuentra la velocidad y la distancia que caen al final de 5 [seg].

Resp: .

2. Una bola de 6 [lb] es lanzada verticalmente hacia abajo hacia la tierra desde una altura de
1000 [pies] con una velocidad inicial de 6 [pies/seg]. Cuando cae, acta sobre la resistencia
2
del aire que es numricamente igual a [lb], donde v [pies/seg] es la velocidad.
3
a) Cul es la velocidad y la distancia que caen al final de un minuto?
b) Con qu velocidad golpea la bola la tierra?

Resp: .

3
3. Una bola que pesa [lb] se lanza verticalmente hacia arriba desde un punto 6 [pies] sobre
4
la superficie de la tierra con una velocidad inicial de 20 [pies/seg]. A medida que se eleva,
1
acta sobre la resistencia del aire que es numricamente igual a [lb], donde v [pies/seg]
64
es la velocidad. Cmo subir la bola?
Resp: .

4. Un barco que pesa 32000 [tons] comienza de descanso bajo la fuerza de un empuje de hlice
constante de 100000 [lb]. La resistencia en libras es numricamente igual a 8000v [pies/seg].
a) Encuentra la velocidad del barco en funcin del tiempo.
b) Encuentre la velocidad lmite (es decir, el lmite de v cuando t ! ).
1.7 Ejercicios 47

c) Calcula cunto tiempo tarda la nave en alcanzar una velocidad de 80 % de la velocidad


lmite.

Resp: .

5. Un cuerpo de masa 100 [gr] se deja caer del resto hacia la tierra desde una altura de 1000
[mt]. Cuando cae, la resistencia del aire acta sobre ella, y esta resistencia (en newtons) es
proporcional a la velocidad v [mt/seg]. Supongamos que la velocidad lmite es 245 [mt/seg].
a) Encuentre la velocidad y la distancia cadas en el tiempo t [seg].
b) Encuentre el tiempo en que la velocidad es un quinto de la velocidad lmite.

Resp: .

6. Un objeto de masa 100 [gr] se lanza verticalmente hacia arriba desde un punto 60 [cm] sobre
la superficie de la tierra con una velocidad inicial de 150 [cm/seg]. Se eleva brevemente y
luego cae verticalmente a la tierra, tiempo en el cual acta sobre la resistencia del aire que es
numricamente igual a 200v [dinas], donde v [cm/seg] es la velocidad.
a) Encuentra la velocidad 0,1 [seg] despus de lanzar el objeto.
b) Encuentre la velocidad 0,1 [seg] despus de que el objeto deje de subir y empiece a caer.

Resp: .

7. Dos personas estn montando en una lancha y el peso combinado de individuos, motor,
barco, y equipo es 640 [lb]. El motor ejerce una fuerza constante de 20 [lb] en el barco en la
direccin del movimiento, mientras que la resistencia (en libras) es numricamente igual a
una y media veces la velocidad (en pies por segundo). Si la embarcacin comenz a partir
del reposo, encuentre la velocidad de la tabla despus de (a) 20 [seg], (b) 1 [min].
Resp: .

8. Un barco que pesa 150 [lb] con una persona que pesa 170 [lb] est siendo remolcado en una
direccin determinada a un ritmo de 20 [mph]. En el momento t = 0, la cuerda de remolque
se suelta repentinamente y la persona comienza a remar en la misma direccin. La resistencia
(en libras) es numricamente igual a dos veces la velocidad (en pies por segundo).
a) Encuentre la velocidad del barco 15 [seg] despus de que la cuerda de remolque fuera
rechazada.
b) Cuntos segundos despus de que la cuerda de remolque sea desechada, la velocidad
ser la mitad de la que se estaba remolcando el barco?

Resp: .

9. Una bala que pesa 1 [onz] se dispara verticalmente hacia abajo desde un helicptero esta-
cionario con una velocidad de boca de 1200 [pies/seg]. la resistencia del aire (en libras) es
numricamente igual a 16 5 v2 , donde v [pies/seg] es la velocidad. Encuentra la velocidad de
la bala en funcin del tiempo.
Resp: .

10. Una concha que pesa 1 [lb] se dispara verticalmente hacia arriba desde la superficie de la
tierra con una velocidad de boca de 1000 [pies/seg]. La resistencia del aire (en libras) es
numricamente igual a 10 4 v2 , donde v [pies/seg] es la velocidad.
48 Captulo 1. Aplicaciones

a) Encuentre la velocidad de la concha ascendente como una funcin del tiempo.


b) Cunto tiempo subir la concha?

Resp: .

11. Un objeto que pesa 16 [lb] se deja caer del resto en la superficie de un lago tranquilo y despus
empieza a hundirse. Mientras que su peso tiende a forzarlo hacia abajo, la flotabilidad del
objeto tiende a forzarlo hacia atrs hacia arriba. Si esta fuerza de flotabilidad es de 6 [lb]
y la resistencia del agua (en libras) es numricamente igual a dos veces el cuadrado de la
velocidad (en pies por segundo), encuentre la frmula para la velocidad del objeto hundido
como una funcin del tiempo.
Resp: .

12. Un objeto que pesa 12 [lb] se coloca debajo de la superficie de un lago tranquilo. La flotabili-
dad del objeto es 30 [lb]; debido a esto el objeto comienza a elevarse. Si la resistencia del
agua (en libras) es numricamente igual al cuadrado de la velocidad (en pies por segundo)
y la superficie del objeto en 5 [seg], encuentre la velocidad del objeto en el instante en que
alcance la superficie.
Resp: .

13. Un hombre est empujando un trineo cargado a travs de un campo llano de hielo a la
velocidad constante de 10 [pies/seg]. Cuando el hombre est a medio camino en el campo de
hielo, deja de empujar y deja que el trineo cargado contine. El peso combinado del trineo y
3
su carga es 80 [lb]; la resistencia del aire (en libras) es numricamente igual a v, donde v es
4
la velocidad del trineo (en pies por segundo); y el coeficiente de friccin de los corredores en
el hielo es 0,04. Hasta qu punto el trineo seguir movindose despus de que el hombre
deje de empujar?
Resp: .

14. Una nia en su trineo acaba de deslizarse por una colina en un nivel de hielo y est empezando
a disminuir la velocidad. En el instante en que su velocidad es 5 [pies/seg], el padre de la nia
se ejecuta y comienza a empujar el trineo hacia adelante, ejerciendo una fuerza constante de
15 [lb] en la direccin del movimiento. El peso combinado de la nia y el trineo es de 96
[lb], la resistencia del aire (en libras) es numricamente igual a la mitad de la velocidad (en
pies por segundo), y el coeficiente de friccin de los corredores sobre el hielo es 0,05. Qu
tan rpido se mueve el trineo 10 [seg] despus de que el padre comienza a empujar?
Resp: .

15. Un caso de leche enlatada que pesa 24 [lb] se libera del resto en la parte superior de una
diapositiva plana de metal que es 30 [pies] de largo e inclinado 45 a la horizontal. La
resistencia del aire (en libras) es numricamente igual a un tercio de la velocidad (en pies por
segundo) y el coeficiente de friccin es 0,4.
a) Cul es la velocidad del caso en movimiento 1 [seg] despus de que se libera?
b) Cul es la velocidad cuando el caso alcanza la parte inferior de la diapositiva?

Resp: .

16. Un muchacho va en trineo por una larga pendiente de 30 . El peso combinado del nio y su
trineo es 72 [lb] y la resistencia del aire (en libras) es numricamente igual al doble de su
1.7 Ejercicios 49

velocidad (en pies por segundo). Si comenzaron a partir del reposo y su velocidad al final de
5 [seg] es 10 [pies/seg], cul es el coeficiente de friccin de los corredores de trineo en la
nieve?
Resp: .

17. Un objeto que pesa 32 [lb] se libera del resto 50 [pies] por encima de la superficie de un lago
tranquilo. Antes de que el objeto alcance la superficie del lago, la resistencia del aire (en
libras) est dada por 2v, donde v [pies/seg] es la velocidad. Despus de que el objeto pasa por
debajo de la superficie, la resistencia al agua (en libras) est dada por 6v. Adems, el objeto
es entonces estimulado por una fuerza de flotabilidad de 8 [lb]. Encuentre la velocidad del
objeto 2 [seg] despus de pasar por debajo de la superficie del lago.
Resp: .

18. Un cohete de masa m se dispara verticalmente hacia arriba desde la superficie de la tierra con
una velocidad inicial v = v0 . La nica fuerza en el cohete que consideramos es la atraccin
gravitacional de la tierra. Entonces, de acuerdo con la ley de gravitacin de Newton, la
k
aceleracin a del cohete est dada por a = , donde k > 0 es una constante de propor-
x2
cionalidad y x es la distancia hacia arriba"desde el centro de la tierra a lo largo de la lnea
de movimiento. En el tiempo t = 0, x = R (donde R es el radio de la tierra), a = g (donde
dv
g es la aceleracin debida a la gravedad), y v = v0 . Exprese a = como en la ecuacin
dt
dv
mv = F, aplique los datos iniciales apropiados y observe que v satisface la ecuacin
dx
dv gR2
diferencial v = . Resuelve esta ecuacin diferencial, aplica la condicin inicial
dx x2
apropiada y expresa v como una funcin de x. En pparticular, muestre que el valor mnimo de
v0 para el cual el cohete escapar de la tierra es 2gR. Esta es la llamada velocidad de esca-
pe; y usando R = 4000 [mi], g = 32 [pies/seg2 ], se encuentra que esto es aproximadamente
25000 [mph] (o 7 [mi/seg]).
Resp: .

19. Un cuerpo de masa m est en movimiento rectilneo a lo largo de un eje horizontal. La fuerza
resultante que acta sobre el cuerpo est dada por kx, donde k > 0 es una constante de
proporcionalidad y x es la distancia a lo largo del eje desde un punto fijo O. El cuerpo tiene
una velocidad inicial v = v0 cuando x = x0 . Aplicar la segunda ley de Newton en la forma
dv dv
mv dx = F y as escribir la ecuacin diferencial de movimiento en la forma mv dx = kx.
Resuelve la ecuacin diferencial, aplica la condicin inicial y as expresa el cuadrado de la
velocidad v en funcin de la distancia x. Recordando que v = dx dt , muestra que la relacin
entre v y x as obtenida se cumple para todo el tiempo t por
s r !
mv 2
k
x = x02 + 0 sin t +f ,
k m

donde f es una constante.


Resp: .

20. Un objeto que pesa 4 [lb] se lanza verticalmente hacia abajo hacia la tierra desde una altura
de 2000 [pies] con una velocidad inicial de 20 [pies/seg]. Al caer, acta sobre la resistencia
del aire que es numricamente igual a 2v [lb], donde v es la velocidad (en pies por segundo).
a) Cul es la velocidad y la distancia que caen al final de 1 segundos?
50 Captulo 1. Aplicaciones

b) Con qu velocidad el objeto golpea la tierra?

Resp: .

21. Una persona est montando en una lancha, y el peso combinado de la persona, el motor, el
barco y el equipo es 480 [lb]. El motor ejerce una fuerza constante de 16 [lb] en el barco en
la direccin del movimiento, mientras que la resistencia (en libras) es numricamente igual al
cuadrado de la velocidad (en pies por segundo). Supongamos que el barco sali del descanso.
Cul es la velocidad del barco despus de (a) 2 segundos? (b) 5 segundos?
Resp: .

22. Un pedazo de madera que pesa 160 [lb] es empujado de la parte superior de una diapositiva
plana que es de 50 [pies] de largo e inclinado 30 a la horizontal. La resistencia del aire (en
libras) es numricamente igual a la velocidad (en pies por segundo), el coeficiente de friccin
es 0,5, y la madera empieza a deslizarse con velocidad inicial 8 [pies / seg].
a) Qu tan rpido se mueve la madera 2 segundos despus de que comience a deslizarse?
b) A qu distancia se ha deslizado 2 segundos despus de que comienza a deslizarse?

Resp: .

1.8 Problemas de tasa


En ciertos problemas, la velocidad a la cual cambia una cantidad es una funcin conocida de la
cantidad presente y/o del tiempo, y se desea encontrar la cantidad misma. Si x denota la cantidad de
la cantidad presente en el momento t, entonces dxdt denota la velocidad a la cual cambia la cantidad
y nos conduce a una ecuacin diferencial. En esta seccin consideramos ciertos problemas de este
tipo.

1.8.1 Tasa de crecimiento y decrecimiento


Ejemplo 1.17 La velocidad a la que los ncleos radiactivos se desintegran es proporcional al
nmero de tales ncleos que estn presentes en una muestra dada. La mitad del nmero original de
ncleos radiactivos ha sufrido desintegracin en un perodo de 1500 aos.
1. Qu porcentaje de los ncleos radiactivos originales permanecer despus de 4500 aos?
2. En cuntos aos quedar slo una dcima parte del nmero original?
Solucin Sea x la cantidad de ncleos radiactivos presentes despus de t aos. Entonces dx dt
representa la velocidad a la que los ncleos se descomponen. Ya que los ncleos se descomponen a
una tasa proporcional a la cantidad presente, tenemos

dx
= Kx, (1.80)
dt

donde K es una constante de proporcionalidad. El monto x es claramente positivo; adems, dado


que x est disminuyendo, dx dt < 0. As, a partir de la ecuacin (1.80), debemos tener K < 0. Con
el fin de enfatizar que x est disminuyendo, preferimos reemplazar K por una constante positiva
precedida por un signo menos. As, hacemos que k = K > 0 y escribamos la ecuacin diferencial
(1.80) en la forma

dx
= kx. (1.81)
dt
1.8 Problemas de tasa 51

Denotando x0 la cantidad inicialmente presente, tambin tenemos la condicin inicial

x(0) = x0 . (1.82)

Sabemos que necesitaremos tal condicin para determinar la constante arbitraria que aparecer en
una familia de soluciones de un parmetro de la ecuacin diferencial (1.81). Sin embargo, aparen-
temente necesitaremos algo ms, porque la ecuacin (1.81) contiene una constante desconocida
de proporcionalidad k. Esta otra cosa aparece en el enunciado del problema, pues se nos dice que
la mitad del nmero original se deshace en 1500 aos. As, la mitad tambin permanece en ese
momento, y esto da a la vez la condicin
1
x(1500) = x0 . (1.83)
2
La ecuacin diferencial (1.81) es claramente separable; separando variables, integrar y simplificar,
tenemos a la vez
kt
x = ce .

Aplicando la condicin inicial (1.82), x = x0 cuando t = 0, encontramos que c = x0 y por lo tanto


obtenemos
kt
x = x0 e . (1.84)
1
Todava no hemos determinado k. As, ahora aplicamos la condicin (1.83), x = 2 x0 cuando
t = 1500, a la ecuacin (1.84). Encontramos
1 1500k
x0 = x0 e ,
2
o
1
(e k )1500 = ,
2
o finalmente
1500
1
k 1
e = .
2
De esto encontramos
ln 2
k= 0,00046.
1500
Usando esto, (1.84) se convierte en
0,00046t
x = x0 e . (1.85)

Alternativamente, observamos que en realidad no necesitamos k en s mismo (1.84), sino ms bien


slo e k , que ya hemos obtenido. Encontramos
1500
1
k 1
e = ,
2
y ahora sustituimos esto por (1.84) para obtener
" 1 #t
k t 1 1500
x = x0 (e ) = x0 ,
2
52 Captulo 1. Aplicaciones

o
1500
t
1
x = x0 . (1.86)
2
Cada una de las dos expresiones equivalentes (1.85) y (1.86) da el nmero x de ncleos radiactivos
que estn presentes en el tiempo t. Utilizaremos la frmula (1.86) para responder a las preguntas 1
y 2.
La pregunta 1 nos pide qu porcentaje del nmero original permanecer despus de 4500 aos. por
lo tanto, t = 4500 en la ecuacin (1.86), y encontrar
3
1 1
x = x0 = x0 .
2 8
As, un octavo o 12,5 % del nmero original permanecer despus de 4500 aos. La pregunta 2 nos
1
pide cundo slo quedar una dcima parte. Por lo tanto, hacemos x = 10 x0 en la ecuacin (1.86) y
resolvamos por t. Tenemos
1500
t
1 1
= .
10 2
Usando logaritmos, obtenemos
1500
t
1 1 t 1
ln = ln = ln .
10 2 1500 2
De esto se sigue inmediatamente que

t ln 1
= 10
1500 ln 12
o
1500 ln 10
t= 4985 [aos].
ln 2

Ejemplo 1.18 Un cuerpo se enfra en el aire de temperatura constante Tm = 20 C. Si la tempera-


tura del cuerpo cambia de 100 C a 60 C en 20 [min], determine cunto ms tiempo necesitar para
que la temperatura descienda a 30 C.
Solucin La ley de enfriamiento de Newton requiere que
dT
= k(T Tm ).
dt
La solucin general es
Z Z
dT kt
= k dt + c =) ln |T Tm | = kt + ln |c| =) T = Tm + ce .
T Tm
En t = 0 [min], T = 100 C:
k0
100 = 20 + ce = 20 + c =) c = 80.

En t = 20 [min], T = 60 C:

k20 1 60 20
60 = 20 + 80e =) k= ln = 0,03466.
20 80
1.8 Problemas de tasa 53

Por lo tanto
0,03466 t 1 T 20
T = 20 + 80e =) t= ln [min].
0,03466 80
Cuando T = 30 C:
1 30 20
t= ln = 60 [min].
0,03466 80
Por lo tanto, necesitar otros 60 20 = 40 [min] para que la temperatura descienda a 30 C.

Ejemplo 1.19 La tasa de prdida de calor de un edificio es igual a k1 [TB (t) TA (t)], donde TB (t)
y TA (t) son las temperaturas del edificio y la atmsfera en tiempo t, respectivamente, y k1 es una
constante. La tasa de calor suministrada al edificio por el sistema de calefaccin viene dada por
Q + k2 [TS TB (t)], donde TS es la temperatura establecida del edificio, Q y k2 son constantes. El
valor de Q es tal que el edificio se mantiene a la temperatura establecida cuando la atmsfera est a
temperatura constante T0 . La capacidad trmica del edificio es c.
1. Establezca la ecuacin diferencial que rige la temperatura del edificio TB (t).
2. Si la temperatura atmosfrica flucta sinusoidalmente sobre el valor medio T0 con una amplitud
de T1 C y un perodo de 2pw [hora], es decir,

TA = T0 + T1 sin wt,

determine la amplitud de la variacin de temperatura del edificio debido a la fluctuacin de la


temperatura atmosfrica.
3. Supongamos que T1 = 12 C, kc1 = 4 [hora], la temperatura atmosfrica flucta con un perodo
p
de 24 horas, es decir, w = 12 . Se requiere que la temperatura del edificio permanezca dentro de
3 C del valor establecido, es decir, la amplitud de la fluctuacin de la temperatura es menor o igual
a 3 C. Demuestre que el valor de la relacin kk21 debe satisfacer
r
k2 p2
4 1 1.
k1 144
Solucin 1. Considere un perodo de tiempo de t a t + Dt, el principio de conservacin de
energa requiere

Suministro de calor en el tiempo Dt Prdida de calor en el tiempo Dt = cDTB ,

dnde
Suministro de calor en el tiempo Dt = {Q + k2 [TS TB (t)]}Dt,
Prdida de calor en el tiempo Dt = k1 [TB (t) TA (t)]Dt.
Por lo tanto,

{Q + k2 [TS TB (t)]}Dt k1 [TB (t) TA (t)]Dt = cDTB .

Dividiendo la ecuacin para Dt y tomando el lmite como Dt ! 0 nos conduce a


dTB
Q + k2 [TS TB (t)] k1 [TB (t) TA (t)] = c ,
dt
o

c dTB k2 Q k2
+ 1+ TB (t) = + TS + TA (t).
k1 dt k1 k1 k1
54 Captulo 1. Aplicaciones

2. Dado que TA = T0 + T1 sin wt, la ecuacin diferencial se vuelve



c dTB k2 Q k2
+ 1+ TB (t) = + TS + T0 + T1 sin wt,
k1 dt k1 k1 k1
o

dTB k1 k2 k1 Q k2 k1 T1
+ 1+ TB (t) = + TS + T0 + sin wt,
dt c k1 c k1 k1 c
La ecuacin diferencial es lineal de primer orden de la forma

dTB k1 k2 k1 Q k2 k1 T1
+ p(t)TB (t) = q(t), p(t) = 1+ , q(t) = + TS + T0 + sin wt,
dt c k1 c k1 k1 c
Es fcil de evaluar
R R k1 k

k1

k

R R k1 k

k1

k

p(t) dt 1+ k2 dt 1+ k2 t p(t) dt 1+ k2 dt 1+ k2 t
e =e c 1 =e c 1 , e =e c 1 =e c 1 ,

y
Z Z

R
p(t) dt Q k2 k1 k1 T1 k1 k2
c 1+ k1 t
q(t)e dt = + TS + T0 + sin wt e dt,
k1 k1 c c

a1 a2
= +p sin (wt f ) ekt ,
k k2 + w 2
donde

k1 k2 k1 Q k2 k1 T1 w
k= 1+ , a1 = + TS + T0 , a2 = , f = arctan ,
c k1 c k1 k1 c k
Por lo tanto, la solucin general es
R
Z R
p(t) dt
TB (t) = e q(t)e p(t) dt dt + c

kt a1 a2
= e +p sin (wt f ) ekt + c
k k + w2
2
a1 a2
= +p sin (wt f ) + ce kt .
k k +w
2 2

donde pk2a+w
2
2
sin (wt f ) es la variacin debida a la fluctuacin de la temperatura atmosfrica. La
amplitud de la variacin de temperatura debida a la fluctuacin de la temperatura atmosfrica es
k1 T1
a2 c T1
a= p = rh i2 = r 2 2 .
k + w2
2
k1 k2 k2 cw
c 1+ k1 + w2 1+ k1 + k1

3. Que la amplitud de la fluctuacin de la temperatura es menor o igual a 3 C significa a 3, es


decir,
T1
r 2 2 3,
k2
1 + k1 + cwk1

lo que da
2 2 s 2 2
T1 k2 2 cw k2 T1 cw
1+ + =) 1,
3 k1 k1 k1 3 k1
1.8 Problemas de tasa 55

y
s 2 2 r
k2 12 4p p2
1=4 1 1.
k1 3 12 144

Ejemplo 1.20 La Ley de Enfriamiento de Newton establece que la velocidad de cambio de la


temperatura de un cuerpo de enfriamiento es proporcional a la diferencia entre la temperatura del
cuerpo y la temperatura constante del medio que rodea al cuerpo. Aplicar esta ley al siguiente
problema.
Un cuerpo de temperatura 80 F se coloca en una habitacin de temperatura constante 50 F en
el momento t = 0; y al cabo de 5 minutos, el cuerpo se ha enfriado a una temperatura de 70
F. Determine la temperatura del cuerpo como una funcin del tiempo para t > 0. En particular,
responda las siguientes preguntas:
1. Cul es la temperatura del cuerpo al final de 10 minutos?
2. Cundo ser la temperatura del cuerpo 60 F?
3. Despus de cuntos minutos la temperatura del cuerpo estar dentro de 1 F de la temperatura
constante de 50 F de la habitacin?
Solucin Sea x la temperatura Fahrenheit del cuerpo en el momento t. Segn la Ley de enfria-
miento de Newton, tenemos de inmediato la ecuacin diferencial
dx
= k(x 50), (1.87)
dt
donde k es la constante de proporcionalidad. La temperatura inicial de 80 da la condicin inicial

x(0) = 80; (1.88)

y la temperatura de 70 al final de 5 minutos da la condicin adicional

x(5) = 70. (1.89)

La ecuacin diferencial (1.87) es separable y lineal. Lo resolvemos como una ecuacin separable y
escribimos
dx
= k dt.
x 50
Integrando, encontramos

ln |x 50| = kt + c0 ,

y de esto

|x 50| = cekt .

Ya que x 50, |x 50| = x 50, y as tenemos

x = 50 + cekt . (1.90)

Aplicamos la condicin inicial (1.88) a esto. Hacemos x = 80 y t = 0 en (1.90) para obtener


80 = 50 + c, de los cuales c = 30. As (1.90) se convierte

x = 50 + 30ekt . (1.91)
56 Captulo 1. Aplicaciones

Ahora aplicamos la condicin adicional (1.89) a (1.91). Hacemos x = 70 y t = 5 para obtener


70 = 50 + 30e5k , de los cuales
2
e5k = . (1.92)
3
A partir de esto, k = 15 ln 23 , y luego una calculadora da k 0,08109. Usando esto, (1.91) se
convierte en
0,08109t
x = 50 + 30e . (1.93)
Alternativamente, observamos que no necesitamos k en s mismo en (1.91), sino solamente ek . De
(1.92), tenemos
15
k 2
e = .
3
El uso de esto, (1.91) se convierte en
15
2
x = 50 + 30 . (1.94)
3
Cada una de las expresiones equivalentes (1.93) y (1.94) da la temperatura del cuerpo como una
funcin del tiempo para t > 0. Utilizaremos la frmula (1.94) para responder a las preguntas 1, 2 y
3.
La pregunta 1 pide la temperatura al final de 10 minutos. Por lo tanto, se hace t = 10 en (1.94).
Encontramos
2
2
x = 50 + 30 63,33 F.
3
La pregunta 2 pide cundo la temperatura x ser 60 . Por lo tanto, se hace x = 60 en (1.94) y se
resuelve por t. Tenemos
5t
2
60 = 50 + 30 ,
3
a partir del cual
5t
2 1
= .
3 3
de donde,
!
ln 13
t =5 13,55 [min].
ln 23
La pregunta 3 pide despus de cuntos minutos la temperatura estar dentro de 1 de la temperatura
ambiente constante de 50 . As buscamos el tiempo cuando la temperatura x es 51. De esta manera,
haciendo x = 51 en (1.94), rpidamente encontramos
5t
2 1
= .
3 30
a partir del cual
!
1
ln 30
t =5 41,94 [min].
ln 23
As que en aproximadamente 42 minutos la temperatura del cuerpo estar dentro de 1 de la de la
habitacin.
1.8 Problemas de tasa 57

1.8.2 Crecimiento de la poblacin


A continuacin consideramos el crecimiento de una poblacin (por ejemplo, humana, una
especie animal o una colonia de bacterias) en funcin del tiempo. Obsrvese que una poblacin
en realidad aumenta discontinuamente en cantidades enteras. Sin embargo, si la poblacin es muy
grande, tales incrementos individuales son esencialmente insignificantes en comparacin con toda
la poblacin misma. En otras palabras, el aumento de poblacin es aproximadamente continuo. Por
lo tanto, asumiremos que este aumento es de hecho continuo y, de hecho, que la poblacin es una
funcin continua y diferenciable del tiempo.
Dada una poblacin, hacemos que x sea el nmero de individuos en l en el momento t. Si
asumimos que la tasa de cambio de la poblacin es proporcional al nmero de individuos en ella en
cualquier momento, nos llevan a la ecuacin diferencial

dx
= kx, (1.95)
dt
donde k es una constante de proporcionalidad. La poblacin x es positiva y est aumentando, y por
lo tanto dx
dt > 0. Por lo tanto, de (1.95), debemos tener k > 0. Supongamos ahora que en el tiempo t0
la poblacin es x0 . Entonces, adems de la ecuacin diferencial (1.95) tenemos la condicin inicial

x(t0 ) = x0 . (1.96)

La ecuacin diferencial (1.95) es separable. separando variables, integrando y simplificando,


obtenemos

x = cekt .

Aplicando la condicin inicial (1.96), x = x0 a t = t0 , a esto, tenemos x0 = cekt0 . A partir de esto


encontramos inmediatamente c = x0 e kt0 , obteniendo as la solucin nica

x = x0 ek(t t0 )
(1.97)

de la ecuacin diferencial (1.95) que satisface la condicin inicial (1.96).


De (1.97) vemos que una poblacin gobernada por la ecuacin diferencial (1.95) con k > 0 y
la condicin inicial (1.96) es aquella que aumenta exponencialmente con el tiempo. Esta ley de
crecimiento de la poblacin se llama la ley de Malthusian. Ahora debemos preguntarnos si hay
casos en los que tal modelo de crecimiento demogrfico es realmente realista. En respuesta a esto,
se puede demostrar que este modelo, con un valor adecuado de k, es notablemente preciso en el caso
de la poblacin humana de la tierra durante las ltimas dcadas. Tambin se sabe que es excelente
para ciertas especies de mamferos, con k adecuado, bajo ciertas condiciones de realizacin y para
ciertos perodos de tiempo. Por otra parte, volviendo al caso de la poblacin humana de la tierra, se
puede demostrar que la ley malthusiana resulta ser bastante irrazonable cuando se aplica al futuro
lejano. Tambin es completamente poco realista para otras poblaciones (por ejemplo, colonias de
bacterias) cuando se aplica durante perodos de tiempo suficientemente largos. La razn de esto
no es difcil de ver. Pues, segn (1.97), una poblacin modelada por esta ley siempre aumenta e
incluso lo hace a un ritmo cada vez mayor; mientras que la observacin muestra que una poblacin
dada simplemente no crece indefinidamente.
El crecimiento de la poblacin se representa de manera ms realista en muchos casos suponiendo
que el nmero de individuos x en la poblacin en el momento t se describe mediante una ecuacin
diferencial de la forma
dx
= kx l x2 , (1.98)
dt
58 Captulo 1. Aplicaciones

donde k > 0 y l > 0 son constantes. El trmino adicional l x2 es el resultado de alguna causa que
tiende a limitar el crecimiento final de la poblacin. Por ejemplo, tal causa podra ser insuficiente
espacio vital o suministro de alimentos, cuando la poblacin se hace suficientemente grande.
En cuanto a la eleccin de l x2 para el trmino que representa el efecto de la causa, se puede
argumentar como sigue: Asumiendo que la causa afecta a toda la poblacin de x miembros, entonces
el efecto sobre cualquier individuo es proporcional a x. Por lo tanto, el efecto sobre todos los x
individuos en la poblacin sera proporcional a x x = x2 .
Asumimos as que una poblacin se describe mediante una ecuacin diferencial de la forma
(1.98) con constantes k > 0 y l > 0, y una condicin inicial de la forma (1.96). En la mayora
de estos casos, resulta que la constante l es muy pequea comparada con la constante k. Por lo
tanto, para x suficientemente pequeo, el trmino kx predomina, por lo que la poblacin crece muy
rpidamente por un tiempo. Sin embargo, cuando x se hace suficientemente grande, el trmino
l x2 tiene una influencia comparativamente mayor, y el resultado de esto es una disminucin
en la tasa de crecimiento rpido. Observamos que la ecuacin diferencial (1.98) es a la vez una
ecuacin separable y una ecuacin de Bernoulli. La ley del crecimiento de la poblacin as descrita
se llama ley logstica del crecimiento. Ahora consideramos un ejemplo especfico de este tipo de
crecimiento.
Ejemplo 1.21 La poblacin x de una cierta ciudad satisface la ley logstica

dx 1 1
= x x2 (1.99)
dt 100 (10)8

donde el tiempo t se mide en aos. Dado que la poblacin de esta ciudad es 100000 en 1980,
determine la poblacin en funcin del tiempo por t > 1980. En particular, responda las siguientes
preguntas:
(a) Cul ser la poblacin en 2000?
(b) En qu ao se duplica la poblacin de 1980?
(c) Asumiendo que la ecuacin diferencial (1.99) se aplica para todos los t > 1980, cun grande
ser la poblacin en ltima instancia?
Solucin Debemos resolver la ecuacin diferencial separable (1.99) sujeta a la solucin inicial

x(1980) = 100000. (1.100)

Separando variables en (1.99) obtenemos

dx
2x
= dt
(10) (10) 8 x2

y por lo tanto

dx
= dt
(10) 2 x[1 (10) 6 x]

Usando fracciones parciales, esto se convierte en



1 (10) 6
100 + dx = dt.
x 1 (10) 6 x

Integrando, asumiendo 0 < x < 106 , obtenemos

100{ln x ln[1 (10) 6 x]} = t + c1


1.8 Problemas de tasa 59

y por lo tanto

x 1
ln 6
= t + c2 .
1 (10) x 100
As encontramos
x t

6
= ce 100 .
1 (10) x
Resolviendo esto por x, finalmente obtenemos
t
ce 100
x= t . (1.101)
1 + (10) 6 ce 100
Ahora aplicando la condicin inicial (1.100) a esto, tenemos
ce19,8
(10)5 = ,
1 + (10) 6 ce19,8
de donde obtenemos
(10)5 (10)6
c= = .
e19,8 [1 (10)5 (10) 6 ] 9e19,8
Sustituyendo este valor por c de nuevo en (1.101) y simplificando, obtenemos la solucin en la
forma
(10)6
x= t . (1.102)
1 + 9e19,8 100

Esto da a la poblacin x como una funcin del tiempo para t > 1980.
Ahora consideramos las preguntas (a), (b) y (c) del problema. Pregunta (a) pide la poblacin en
el ao 2000. As hacemos t = 2000 en (1.102) y obtenemos
(10)6
x= 119,495.
1 + 9e 0,2
La pregunta (b) pide el ao en que la poblacin duplica. Por lo tanto, se hace x = 200000 = 2(10)5
en (1.102) y se resuelve por t. Tenemos
(10)6
2(10)5 = t ,
1 + 9e19,8 100

a partir del cual


t 4
e19,8 100 = ,
9
y por lo tanto

t 2061.

La pregunta (c) pregunta cun grande ser la poblacin en ltima instancia, suponiendo que la
ecuacin diferencial (1.99) se aplica para todos los t > 1980. Para responder a esto, evaluamos
lm x como t ! usando la solucin (1.102) de (1.99). Encontramos
(10)6
lm x = lm 19,8 t = (10)6 = 1,000,000.
t! t! 1 + 9e 100


60 Captulo 1. Aplicaciones

1.8.3 Problemas de mezcla


Ahora consideramos los problemas de velocidad que involucran mezclas. Una sustancia S se deja
fluir en una cierta mezcla en un recipiente a una cierta velocidad, y la mezcla se mantiene uniforme
por agitacin. Adems, en una de tales situaciones, esta mezcla uniforme fluye simultneamente
fuera del recipiente a otra velocidad (generalmente diferente); en otra situacin tal vez no sea el
caso. En cualquier caso, buscamos determinar la cantidad de la sustancia S presente en la mezcla
en el tiempo t.
Haciendo que x denote la cantidad de S presente en el momento t, la derivada dx dt denota la tasa
de cambio de x con respecto a t. Si IN denota la velocidad a la que S entra en la mezcla y OUT la
velocidad a la que sale, tenemos a la vez la ecuacin bsica

dx
= IN - OUT (1.103)
dt
para determinar la cantidad x de S en el momento t.
Ejemplo 1.22 Un tanque contiene inicialmente 50 [gal] de agua pura. Comenzando en el tiempo
t = 0 una salmuera que contiene 2 [lb] de sal disuelta por galn fluye en el tanque a una velocidad
de 3 [gal/min]. La mezcla se mantiene uniforme por agitacin y la mezcla bien agitada fluye
simultneamente fuera del tanque a la misma velocidad.
1. Cunta sal es el tanque en cualquier momento t > 0?
2. Cunta sal est presente al final de 25 [min]?
3. Cunta sal est presente despus de mucho tiempo?
Solucin Denotamos con x la cantidad de sal en el tanque en el momento t. Aplicamos la ecuacin
bsica (1.103). La salmuera fluye a una velocidad de 3 [gal/min], y cada galn contiene 2 [lb] de
sal. As

IN = (2 [lb/gal])(3 [gal/min]) = 6 [lb/min].

Dado que la tasa de flujo de salida es igual a la tasa de flujo de entrada, el tanque contiene 50
galones de la mezcla en cualquier momento t. Este 50 [gal] contiene x [lb] de sal en el momento t,
1
y as la concentracin de sal en el tiempo t es 50 x [lb/gal]. Por lo tanto, como la mezcla fluye a una
velocidad de 3 [gal/min], tenemos
x 3x
OUT = [lb/gal] (3 [gal/min]) = [lb/min].
50 50
Por lo tanto, la ecuacin diferencial para x en funcin de t es

dx 3x
=6 . (1.104)
dt 50
Dado que inicialmente no haba sal en el tanque, tambin tenemos la condicin inicial

x(0) = 0. (1.105)

La ecuacin (1.104) es lineal y separable. Separando variables, tenemos

dx 3
= dt.
100 x 50
Integrando y simplificando, obtenemos
3t
x = 100 + ce 50 .
1.8 Problemas de tasa 61

Al aplicar la condicin (1.105), x = 0 a t = 0, encontramos que c = 100. As tenemos


3t

x = 100 1 e 50 . (1.106)

Esta es la respuesta a la pregunta 1. En cuanto a la pregunta 2, al final de 25 [min], t = 25, y la


ecuacin (1.106) da
1,5
x(25) = 100(1 ce ) 78 [lb].

La pregunta 3 esencialmente nos pregunta cunta sal est presente cuando t ! . Para responder a
esto, reemplazamos t ! en la ecuacin (1.106) y observamos que x ! 100.

Ejemplo 1.23 Un tanque grande contiene inicialmente 50 [gal] de salmuera en la cual se disuelven
10 [lb] de sal. La salmuera que contiene 2 [lb] de sal disuelta por galn fluye en el tanque a una
velocidad de 5 [gal/min]. La mezcla se mantiene uniforme por agitacin y la mezcla agitada fluye
simultneamente a una velocidad menor de 3 [gal/min]. Cunta sal est en el tanque en cualquier
momento t > 0?
Solucin Sea x la cantidad de sal en el momento t. Nuevamente usaremos la ecuacin (1.103).
Procediendo, tenemos

IN = (2 [lb/gal])(5 [gal/min]) = 10 [lb/min].

tambin una vez ms

OUT = (C [lb/gal]) (3 [gal/min]) = 3C [lb/min].

donde C [lb/gal] denota la concentracin. Pero aqu, dado que la tasa de flujo de salida es diferente
de la del flujo de entrada, la concentracin no es tan simple. En el momento t = 0, el tanque contiene
50 [gal] de salmuera. Dado que la salmuera fluye a una velocidad de 5 [gal/min] pero sale a la
velocidad ms lenta de 3 [gal/min], hay una ganancia neta de 5-3=2 [gal/min] de salmuera en el
tanque. Por lo tanto, al final de t [min] la cantidad de salmuera en el tanque es

50 + 2t [gal].

Por lo tanto, la concentracin en el tiempo t [min] es


x
[lb/gal].
50 + 2t
y entonces
3x
OUT = [lb/min].
50 + 2t
Por lo tanto, la ecuacin diferencial se vuelve
dx 3x
= 10 . (1.107)
dt 50 + 2t
Como inicialmente haba 10 [lb] de sal en el tanque, tenemos la condicin inicial

x(0) = 10. (1.108)

La ecuacin diferencial (1.107) no es separable, pero es lineal. Ponindolo en forma estndar


dx 3
+ x = 10,
dt 50 + 2t
62 Captulo 1. Aplicaciones

encontramos el factor integrador


R 3 3
dt
e 50+2t = (2t + 50) 2 .

Multiplicando a travs de esto, tenemos


3 dx 1 3
(2t + 50) 2 + 3(2t + 50) 2 x = 10(2t + 50) 2 .
dt
o
d h 3
i 3
(2t + 50) 2 x = 10(2t + 50) 2 .
dt
As
3 5
(2t + 50) 2 x = 2(2t + 50) 2 + c

o
c
x = 4(t + 25) + 3 .
(2t + 50) 2
Aplicando la condicin (1.108), x = 10 a t = 0, encontramos
c
10 = 100 + 3
(50) 2
o
3 p
c = (90)(50) 2 = 22,500 2.

Por lo tanto, la cantidad de sal en cualquier momento t > 0 viene dada por
p
22,500 2
x = 4t + 100 3 .
(2t + 50) 2

Ejemplo 1.24 Considere un depsito con un volumen de 8000 millones de pies cbicos ([ft3 ])
y una concentracin de contaminantes inicial de 0,25 %. Se tiene una inyeccin diaria de 500
millones de [ft3 ] de agua con una concentracin de contaminantes de 0,05 % y una salida diaria de
agua con iguales caractersticas perfectamente mezclada en el depsito. Cunto tomar reducir la
concentracin de contaminantes a 0,10 % en el depsito?
Solucin Tenemos los siguientes datos
dx
dt : Tasa de cantidad de soluto en el tiempo
V0 = 8000 millones de [ft3 ] Volumen inicial del depsito
ri = 500 millones [ft3 /da] Velocidad de entrada
r0 = 500 millones [ft3 /da] Velocidad de salida
ci = 0,05 % = 0,0005 Concentracin inicial
Como
dx r0 x dx 500 x
= ri ci ) = 5000 0,0005
dt V0 + (ri r0 ) t dt 8000 + (500 500) t

dx 1 1 dx 1 1
= x ) + x=
dt 4 16 dt 16 4
1.9 Ejercicios 63

1
t
x = 4 + ce 16 , x(0) = 20.

con esta condicin inicial encontramos c = 16


1
t
x = 4 + 16e 16

encontramos el valor de t cuando x = 8


1
t 1
8 = 4 + 16e 16 ) t = 16 ln ) t 22,18 das
4

Ejemplo 1.25 Una pequea barra de metal se saca de un horno cuya temperatura es una cons-
tante de 300 F dentro de una habitacin cuya temperatura es una constante de 70 F. De manera
simultnea, una barra de metal idntica que estaba en la habitacin se coloca en el horno. Asuma
que el tiempo t se mide en minutos. Analice: por qu existe un valor futuro del tiempo, llmelo
t > 0, al cual la temperatura de cada barra es idntica?
Solucin
Z T Z t Z T Z t
dT dT
= k dt y = k dt
300 T 70 0 70 T 300 0

T 300 T 70
ln = k t y ln = k t
230 230

T 70 T 300 T 70 T 300
ln = ln ) = ) T = 185 F
230 230 230 230

1 185 70 1 1
t = ln ) t = ln > 0.
k 230 k 2

Siendo las barras del mismo material y sometidas a los mismos cambios de temperatura en un
tiempo t > 0, entonces en un instante futuro t las barras tendrn iguales temperaturas a T = 185 F,
con una misma proporcin k.

1.9 Ejercicios
1. Supongamos que la velocidad a la que los ncleos radiactivos se desintegran es proporcional
al nmero de tales ncleos que estn presentes en una muestra dada. En una cierta muestra
10 % del nmero original de ncleos radiactivos han sufrido desintegracin en un perodo de
100 aos.
a) Qu porcentaje de los ncleos radiactivos originales permanecer despus de 1000
aos?
b) En cuntos aos slo quedar una cuarta parte del nmero original?

Resp: .
64 Captulo 1. Aplicaciones

2. Una cierta sustancia qumica se convierte en otra sustancia qumica por una reaccin qumica.
La velocidad a la que se convierte el primer producto qumico es proporcional a la cantidad
de este producto qumico presente en cualquier instante. El diez por ciento de la cantidad
original de la primera sustancia qumica se ha convertido en 5 [min].
a) Qu porcentaje de la primera sustancia qumica se habr convertido en 20 [min]?
b) En cuntos minutos se habrn convertido 60 % de la primera sustancia qumica?

Resp: .

3. Una reaccin qumica convierte un determinado producto qumico en otro producto qumico
y la velocidad a la que se convierte el primer producto qumico es proporcional a la cantidad
de este producto qumico presente en cualquier momento. Al final de una hora, 50 [gm]
del primer producto qumico permanecen; mientras que al final de tres horas, slo 25 [gm]
permanecen.
a) Cuntos gramos de la primera sustancia qumica estaban presentes inicialmente?
b) Cuntos gramos de la primera sustancia qumica permanecern al final de cinco horas?
c) En cuntas horas slo quedar 2 [gm] del primer producto qumico?

Resp: .

4. Una reaccin qumica convierte un determinado producto qumico en otro producto qumico
y la velocidad a la que se convierte el primer producto qumico es proporcional a la cantidad
de este producto qumico presente en cualquier momento. Al final de una hora, dos tercios
[kg] de la primera sustancia qumica permanece; mientras que al final de cuatro horas, slo
queda un tercio [kg].
a) Qu fraccin de la primera sustancia qumica permanece al final de siete horas?
b) Cundo quedar slo una dcima parte de la primera sustancia qumica?

Resp: .

5. En un cierto cultivo de bacterias la tasa de aumento en el nmero de bacterias es proporcional


al nmero presente.
a) Si el nmero se triplica en 5 [hr], cuntos estarn presentes en 10 [hr]?
b) Cundo ser el nmero presente 10 veces el nmero inicialmente presente?

Resp: .

6. Un moho crece a una proporcin que es proporcional a la cantidad presente. Inicialmente


hay 3 [onz] de este moho, y 10 horas ms tarde hay 5 [onz].
a) Cunto moho hay al final de 1 da?
b) Cundo hay 10 [onz] del moho?

Resp: .

7. Supongamos que la ley de enfriamiento de Newton resuelve el siguiente problema: Un cuerpo


de temperatura 100 F se coloca en el momento t = 10 en un medio cuya temperatura se
mantiene en 40 F. Al final de 10 [min], el cuerpo se ha enfriado a una temperatura de 90 F.
a) Cul es la temperatura del cuerpo al final de 30 minutos?
b) Cundo ser la temperatura del cuerpo 50 F?
1.9 Ejercicios 65

Resp: .

8. Supongamos que la Ley de enfriamiento de Newton resuelve el siguiente problema: Un


cuerpo se enfra de 60 C a 50 C en 15 minutos en el aire que se mantiene a 30 C. Cunto
tiempo tardar este cuerpo en enfriarse de 100 C a 80 C en aire que se mantiene a 50 C?
Resp: .

9. Una empanada caliente se toma directamente de un horno y se coloca al aire libre en una
mesa de porche para que se enfre en un da en que la temperatura exterior circundante es
constante 80 F. La temperatura de la empanada era 350 F en el instante t = 0 cuando se
coloc sobre la mesa, y fue 300 F, 5 minutos ms tarde.
a) Cul fue la temperatura 10 minutos despus de que fue colocado en la mesa?
b) Cundo fue su temperatura 100 F?

Resp: .

10. A 10 A.M. una mujer tom una taza de caf instantneo caliente de su horno de microondas y
lo coloc en un contador de cocina cercano para refrescarse. En este instante, la temperatura
del caf era 180 F, y 10 minutos ms tarde era 160 F. Supongamos que la temperatura
constante del caf era 70 F.
a) Cul era la temperatura del caf a las 10 : 15 A.M.?
b) La mujer de este problema le gusta beber caf cuando su temperatura est entre 130 F
y 140 F. Entre qu tiempos debi haber bebido el caf de este problema?

Resp: .

11. Suponga que la poblacin de una determinada ciudad aumenta a una tasa proporcional al
nmero de habitantes en cualquier momento. Si la poblacin duplica en 40 aos, en cuntos
aos se triplicar?
Resp: .

12. La poblacin de una determinada ciudad aumenta a una tasa proporcional al nmero de sus
habitantes presentes en cualquier momento t. Si la poblacin en dicha ciudad era 30000 en
1970 y 35000 en 1980, cul ser la poblacin de Bindville en 1990?
Resp: .

13. La poblacin de roedores de una isla aislada aumenta a una tasa proporcional al nmero de
roedores presentes en cualquier momento t. Si hay x0 roedores en la isla en el tiempo t = 0
y el doble en el tiempo T > 0, cuntos roedores habr en (a) tiempo 2T , (b) tiempo 3T , (c)
tiempo nT , donde n es un entero positivo.
Resp: .

14. Supongamos que la tasa de cambio de la poblacin humana de la tierra es proporcional al


nmero de personas en la tierra en cualquier momento, y supongamos que esta poblacin
est aumentando a razn de 2 % por ao. El almanaque mundial de 1979 da la estimacin de
la poblacin mundial de 1078 como 4219 millones; supongamos que esta cifra es de hecho
correcta.
a) Usando estos datos, expresamos la poblacin humana de la tierra como una funcin del
66 Captulo 1. Aplicaciones

tiempo.
b) De acuerdo con la frmula de la parte (a), cul fue la poblacin de la tierra en 1950?
El almanaque mundial de 1979 da la estimacin de la poblacin mundial de 1950 como
2510 millones. Suponiendo que esta estimacin es casi correcta, comente la exactitud
de la frmula de la parte (a) en el control de estas poblaciones pasadas.
c) De acuerdo con la frmula de la parte (a), cul ser la poblacin de la tierra en 2000?
Esto parece razonable?
d) De acuerdo con la frmula de la parte (a), cul fue la poblacin de la tierra en 1900?
El almanaque mundial de 1970 da la estimacin de la poblacin mundial de 1900 como
1600 millones. Suponiendo que esta estimacin es muy correcta de nuevo, comentar la
exactitud de la frmula de la parte (a) en el control de esas poblaciones pasadas.
e) De acuerdo con la frmula de la parte (a), cul ser la poblacin de la tierra en 2100?
Esto parece razonable?

Resp: .

15. La poblacin humana de una cierta isla satisface la logstica dx 2


dt = kx l x con k = 0,03,
8
l = 3 10 , y el tiempo t medido en aos.
a) Si la poblacin en 1980 es 200000, encuentre una frmula para la poblacin en aos
futuros.
b) De acuerdo con la frmula de la parte (a), cul ser la poblacin en el ao 2000?
c) Cul es el valor lmite de la poblacin en t ! ?

Resp: .

16. Este es un problema general sobre la ley logstica del crecimiento. Una poblacin satisface la
ley logstica dx 2
dt = kx l x y tiene x0 miembros en el momento t0 .
a) Resuelve la ecuacin diferencial dx 2
dt = kx l x y as expresa la poblacin x en funcin
de t.
b) Demuestre que como t ! , la poblacin x se aproxima al valor lmite lk .
c) Demuestre que dx k k
dt aumenta si x < 2l y decrece si x > 2l .
d) Grafique x como una funcin de t para t > t0 .
e) Interpretar los resultados de las partes (b), (c) y (d).

Resp: .

17. La poblacin humana de una cierta isla pequea satisface la ley logstica dx 2
dt = kx l x , con
1 8
k = 400 , l = 10 , y t medidos en aos, siempre y cuando la emigracin anual de la isla sea
descuidada. Sin embargo, el hecho es que cada ao 100 personas se desencantan con la vida
de la isla y se mueven de la isla al continente. Modificar la ecuacin diferencial logstica
dx 2
dt = kx l x con el k dado y l para incluir la emigracin anual indicada. Suponiendo que la
poblacin en 1980 es 20000, resuelva el problema resultante de valor inicial y as encontrar
la poblacin de la isla en funcin del tiempo.
Resp: .

18. Bajo circunstancias naturales la poblacin de ratones en una isla determinada aumentara a
una tasa proporcional al nmero de ratones presentes en cualquier momento, siempre que la
isla no tuviera gatos. No hubo gatos en la isla desde el principio de 1970 hasta el comienzo
de 1980, y durante este tiempo la poblacin de ratones se duplic, alcanzando un mximo
1.9 Ejercicios 67

histrico de 100000 al principio de 1980. En este momento la gente de la isla, alarmada por
el creciente nmero de ratones, import un nmero de gatos para matar a los ratones. Si la
tasa natural indicada de aumento de ratones fue compensada con el trabajo de los gatos, que
mataron 1000 ratones al mes, cuntos ratones permanecieron al principio de 1981?
Resp: .

19. Se dice que una cantidad de dinero invertido atrae intereses compuestos continuamente si la
cantidad de dinero aumenta a una tasa proporcional a la cantidad presente. Supongamos que
$1000 se invierte y atrae intereses compuestos continuamente, donde la tasa de inters anual
es 6 %.
a) Cunto dinero estar presente 10 aos despus de que se invirti la cantidad original?
b) Cunto tiempo tardar la cantidad original de dinero en duplicar?

Resp: .

20. Supongamos que una cierta cantidad de dinero se invierte y atrae inters compuesto conti-
nuamente.
a) Si la cantidad original se duplica en dos aos, cul es la tasa de inters anual?
b) Si la cantidad original aumenta 50 % en seis meses, cunto tardar la cantidad original
en duplicar?

Resp: .

21. Un tanque contiene inicialmente 100 [gal] de salmuera en la cual se disuelve 20 [lb] de sal.
A partir del tiempo t = 0, la salmuera que contiene 3 [lb] de sal disuelta por galn fluye al
tanque a razn de 4 [gal/min]. La mezcla se mantiene uniforme por agitacin y la mezcla
bien agitada fluye simultneamente fuera del tanque a la misma velocidad.
a) Cunta sal est en el tanque al final de 10 minutos?
b) Cundo hay 160 [lb] de sal en el tanque?

Resp: .

22. Un tanque grande contiene inicialmente 100 [gal] de salmuera en el que 10 [lb] de sal se
disuelve. A partir de t = 0, el agua pura fluye hacia el tanque a razn de 5 [gal/min]. La
mezcla se mantiene uniforme por agitacin y la mezcla bien agitada fluye simultneamente a
la velocidad ms lenta de 2 [gal/min].
a) Cunta sal est en el tanque al final de 15 [min] y cul es la concentracin en ese
momento?
b) Si la capacidad del tanque es de 250 [gal], cul es la concentracin en el instante en
que el tanque se desborda?

Resp: .

23. Un tanque contiene inicialmente 100 [gal] de agua pura. A partir de t = 0, una salmuera que
contiene 4 [lb] de sal por galn fluye al tanque a razn de 5 [gal/min]. La mezcla se mantiene
uniforme por agitacin y la mezcla bien agitada sale a la velocidad ms lenta de 3 [gal/min].
a) Cunta sal est en el tanque al final de 20 [min]?
b) Cundo hay 50 [lb] de sal en el tanque?
68 Captulo 1. Aplicaciones

Resp: .

24. Un tanque grande contiene inicialmente 200 [gal] de salmuera en el que 15 [lb] de sal se
disuelve. A partir de t = 0, la salmuera que contiene 4 [lb] de sal por galn fluye al tanque a
una tasa de 3,5 [gal/min]. La mezcla se mantiene uniforme por agitacin y la mezcla bien
agitada sale del tanque a razn de 4 [gal/min].
a) Cunta sal est en el tanque al final de una hora?
b) Cunta sal est en el tanque cuando el tanque contiene slo 50 [gal] de salmuera?

Resp: .

25. Un tanque de 500 litros contiene inicialmente 300 litros de lquido en el que se disuelve
50 [gm] de un determinado producto qumico. El lquido que contiene 30 [gm] por litro de
producto qumico disuelto fluye hacia el tanque a razn de 4 [lit/min]. La mezcla se mantiene
uniforme por agitacin y la mezcla agitada fluye simultneamente a razn de 2,5 [lit/min].
Cunto del producto qumico est en el tanque en el instante en que desborda?
Resp: .

26. Un tanque de 200 litros est inicialmente lleno de lquido en el que se disuelve 40 [gm] de
un determinado producto qumico. El lquido que contiene 50 [gm] por litro de este producto
qumico fluye al tanque a razn de 5 [lit/min]. La mezcla se mantiene uniforme por agitacin
y la mezcla agitada fluye simultneamente a razn de 7 [lit/min]. Cunto del producto
qumico est en el tanque cuando est medio lleno?
Resp: .

27. El aire en una habitacin cuyo volumen es 10000 [pie3 ] prueba 0,15 % de dixido de carbono
se admite a razn de 5000 [pie3 /min].
a) Cul es el porcentaje de dixido de carbono en el aire en la habitacin despus de 3
[min]?
b) Cundo el aire en la habitacin prueba 0,1 % dixido de carbono.

Resp: .

28. El aire en una habitacin 50 [pies] por 20 [pies] por 8 [pies] prueba 0,2 % dixido de carbono.
A partir de t = 0, las pruebas de aire exterior 0, 05 % de dixido de carbono se admite en la
sala. Cuntos pies cbicos de este aire exterior deben ser admitidos por minuto para que el
aire en la habitacin pruebe 0,1 % al final de 30 [min]?
Resp: .

29. Un nuevo producto til se introduce en una poblacin fija aislada de 1000000 personas, y
100 de estas personas adoptan este producto inicialmente, es decir, en el momento t = 0. La
tasa de adopcin del producto es proporcional al nmero de personas que lo han adoptado ya
multiplicado por el nmero de personas que an no lo han hecho. Si hacemos que x denote
el nmero de personas que han adoptado el producto en el momento t, medido en semanas,
entonces tenemos el problema de valor inicial
dx
= kx(1000000 x), x(0) = 100,
dt
donde k es la constante de proporcionalidad.
a) Resuelva este problema de valor inicial.
1.9 Ejercicios 69

b) Cuntas personas han adoptado el producto despus de dos semanas?


c) Cundo la mitad de la poblacin dada la habr adoptado?

Resp: .

30. Exactamente una persona en una poblacin aislada en una isla de 10000 personas viene con
una cierta enfermedad en un da determinado. Supongamos que la tasa a la cual esta enferme-
dad se propaga es proporcional al producto del nmero de personas que tienen la enfermedad
y el nmero de personas que an no la tienen. Si 50 personas tienen la enfermedad despus
de 5 das, cuntas lo tienen despus de 10 das?
Resp: .

31. Dos productos qumicos c1 y c2 reaccionan para formar un tercer producto qumico c3 . La
tasa de cambio del nmero de libras de c3 formada es proporcional a las cantidades de c1 y
c2 presentes en cualquier instante. La formacin de c3 requiere 3 [lb] de c2 por cada libra de
c1 . Suponga que inicialmente hay 10 [lb] de c1 y 15 [lb] de c2 presentes, y que 5 [lb] de c3 se
forman en 15 minutos.
a) Encuentre la cantidad de c3 presente en cualquier momento.
b) Cuntas libras de c3 estn presentes despus de 1 hora?
Sugerencia: Sea x el nmero de libras de c3 formadas en el tiempo t > 0. La formacin
requiere tres veces ms libras de c2 que c1 , por lo que para formar x [lb] de c3 , 3x
4 [lb] de c2 y
x
4 [lb] de c1 son necesarios. Por lo tanto, a partir de las cantidades iniciales dadas, hay 10 4x
[lb] de c1 y 15 3x 4 [lb] de c2 presentes en el momento t cuando x [lb] de c3 se forman. As
tenemos la ecuacin diferencial

dx x 3x
= k 10 15 ,
dt 4 4
donde k es la constante de proporcionalidad. Tenemos la condicin inicial x(0) = 0 y la
condicin adicional x(15) = 5.
Resp: .

32. La tasa a la cual una determinada sustancia se disuelve en el agua es proporcional al producto
de la cantidad no disuelta y la diferencia c1 c2 , donde c1 es la concentracin en la solucin
saturada y c2 es la concentracin en la solucin real. Si est saturada, 50 [gm] de agua
disolvera 20 [gm] de la sustancia. Si 10 [gm] de la sustancia se coloca en 50 [gm] de agua y
la mitad de la sustancia se disuelve en 90 [min], cunto se disolver en 3 horas?
Resp: .

33. Un moho crece a una proporcin que es proporcional a la cantidad presente. En 24 horas la
cantidad de ha crecido de 2 gramos a 3 gramos. Cuntos gramos de ella estn presentes al
final de 24 horas ms?
Resp: .

34. Supongamos que la velocidad a la que los ncleos radiactivos se desintegran es proporcional
al nmero de tales ncleos que estn presentes en una muestra dada. En una cierta muestra,
una cuarta parte del nmero original de ncleos radiactivos ha sufrido una desintegracin en
un perodo de 500 aos.
a) Qu fraccin de los ncleos originales permanecer despus de 1000 aos?
b) En cuntos aos quedar la mitad del nmero original?
70 Captulo 1. Aplicaciones

Resp: .

35. Se retira una olla de agua caliente de la estufa y se coloca en la parte trasera para que se
enfre. En este instante, la temperatura del agua era 200 F, y cinco minutos despus era 190
F. Suponiendo que la ley de enfriamiento de Newton se aplica y que la temperatura que rodea
el recipiente del agua de enfriamiento es 60 , cul ser la temperatura del agua 20 minutos
despus de que se haya enfriado?
Resp: .

36. La poblacin humana de una pequea isla del Pacfico satisface la ley logstica dx
dt = kx l x
2
7
con k = 0,04, l = 2 10 , y el tiempo t medido en aos. La poblacin al comienzo de 1980
es 50000.
a) Encuentre una frmula para la poblacin en aos futuros.
b) De acuerdo con la frmula de la parte (a), cul ser la poblacin al comienzo de 2000?
c) Cundo duplicar la poblacin?

Resp: .

37. Un tanque contiene inicialmente 300 [gal] de salmuera en el que 20 [lb] de sal se disuelve. A
partir de t = 0, la salmuera que contiene 3 [lb] de sal por galn fluye al tanque a razn de 3
[gal/min]. La mezcla se mantiene uniforme por agitacin, y la mezcla bien agitada sale del
tanque a razn de 5 [gal/min]. Cunta sal est en el tanque al final de 15 minutos?
Resp: .

Potrebbero piacerti anche